You are on page 1of 70

CASE KEYWORDS DOCTRINE SUMMARY FACTS ISSUE HELD

PASEI v. DRILON Female domestic Police power is the state Petitioner is a firm engaged W/N the police NO. The Court finds, finally, the impugned guidelines to be
GR No. L-81958; workers authority to enact legislation principally in the recruitment of power was in applicable to all female domestic overseas workers. That it does not
June 30, 1988 that may interfere with Filipino workers, male and female, violation of equal apply to "all Filipina workers" is not an argument for
personal liberty or property in for overseas placement. protection clause unconstitutionality. Had the ban been given universal applicability,
order to promote the general DOLE issued DO No. 1 for the then it would have been unreasonable and arbitrary. For obvious
welfare. temporary suspension of reasons, not all of them are similarly circumstanced. What the
deployment of Filipino domestic Constitution prohibits is the singling out of a select person or group
Its scope, ever-expanding to and household workers. of persons within an existing class, to the prejudice of such a person
meet the exigencies of the Petitioner assailed that it is or group or resulting in an unfair advantage to another person or
times but it may not be discriminatory against males and group of persons. To apply the ban, say exclusively to workers
exercised arbitrarily or females; that it does not apply to deployed by A, but not to those recruited by B, would obviously
unreasonably. all Filipino workers but only to clash with the equal protection clause of the Charter. It would be a
domestic female workers. Ergo, it classic case of what Chase refers to as a law that "takes property
violates equal protection clause. from A and gives it to B." It would be an unlawful invasion of
property rights and freedom of contract and needless to state, an
invalid act.
ICHONG v. Retail Trade Exercise of police power Republic Act 1180 or commonly W/N the Retail NO. The law in question was enacted to remedy a real actual threat
HERNANDEZ Nationalization cannot be bargained away known as ―An Act to Regulate the Trade and danger to national economy posed by alien dominance and
GR No. L-7995; Law through the medium of a Retail Business was passed. The Nationalization control of the retail business and free the citizens and country from
May 31, 1957 treaty or contract said law provides for a prohibition Law is such dominance and control; that the enactment clearly falls within
against foreigners as well as unconstitutional the scope of the police power of the State, thru which and by which
corporations owned by foreigners for it is in conflict it protects its own personality and insures its security and future.
from engaging from retail trade in with treaties which
our country. are generally
Petitioner filed a suit to invalidate accepted
the Retail Trade Nationalization principles of
Law, on the premise that it international law.
violated several treaties which
under the rule of pacta sunt
servanda, a generally accepted
principle of international law,
should be observed by the Court in
good faith.
LUTZ v. ARANETA The Sugar It is inherent in the power to Promulgated in 1940, Section 1 of W/N the YES. The tax is levied with a regulatory purpose, to provide means
GR No. L-7859; Adjustment Act tax that a state be free to CA 567 with a declaration of imposition of taxes for the rehabilitation and stabilization of the threatened sugar
December 22, select the subjects of taxation, emergency, due to the threat to under CA No. 567 industry. In other words, the act is primarily an exercise of the
1955 and it has been repeatedly our industry by the imminent is a valid exercise police power. The protection of a large industry constituting one of
held that "inequalities which imposition of export taxes upon of police power. the great sources of the state's wealth and therefore directly or
result from a singling out of sugar as provided in the Tydings- indirectly affecting the welfare of so great a portion of the
one particular class for McDuffie Act. Wherefore, the population of the State is affected to such an extent by public
taxation or exemption infringe national policy was expressed "to interests as to be within the police power of the sovereign.
no constitutional limitation. obtain a readjustment of the
benefits and to stabilize the sugar As the protection and promotion of the sugar industry is a matter of
industry so as to prepare it for the public concern the Legislature may determine within reasonable
eventuality of the loss of its bounds what is necessary for its protection and expedient for its
preferential position in the United promotion.
States market and the imposition
of the export taxes."

In section 2, Commonwealth Act


567 provides for an increase of the
existing tax on the manufacture of
sugar, on a graduated basis, on
each picul of sugar manufactures;
while section 3 levies on owners or
persons in control of lands
devoted to the cultivation of sugar
cane and ceded to others for a
consideration, on lease or
otherwise.

Plaintiff, Walter Lutz, seeks to


recover from the Collector of
Internal Revenue the sum of
P14,666.40 paid by the estate as
taxes, under section 3 of the Act,
for the crop years 1948-1949 and
1949-1950; alleging that such tax is
unconstitutional and void, being
levied for the aid and support of
the sugar industry exclusively,
which in plaintiff's opinion is not a
public purpose for which a tax may
be constitutionally levied.
LOZANO v. BP 22 The freedom of contract which The constitutionality of Batas W/N BP 22 impairs NO. The gravamen of the offense punished by B.P. 22 is the act of
MARTINEZ is constitutionally protected is Pambansa Bilang 22 (BP 22 for freedom of making and issuing a worthless check or a check that is dishonored
GR No. L-63419; freedom to enter into "lawful" short), popularly known as the contract and not a upon its presentation for payment. It is not the non-payment of an
December 18, contracts. Contracts which Bouncing Check Law, which was valid exercise of obligation which the law punishes. The law is not intended or
1986 contravene public policy are approved on April 3, 1979, is the police power designed to coerce a debtor to pay his debt. The thrust of the law is
not lawful. We must bear in sole issue presented by these to prohibit, under pain of sanctions, the making of worthless checks
mind that checks cannot be petitions for decision. The and putting them is circulation. Because of its deleterious effects on
categorized as mere contracts. question is definitely one of first the public interest, the practice is proscribed by the law. The law
It is a commercial instrument impression in our jurisdiction. The punishes the act not as an offense against property, but an offense
which, in this modern day and language of BP 22 is broad enough against public order.
age, has become a convenient to cover all kinds of checks,
substitute for money; it form whether present dated or The enactment of B.P. 22 is a declaration by the legislature that, as a
part of the banking system and postdated, or whether issued in matter of public policy, the making and issuance of a worthless
therefore not entirely free payment of pre-existing check is deemed a public nuisance to be abated by the imposition of
from the regulatory power of obligations or given in mutual or penal sanctions. The effect of the issuance of a worthless checks
the state. simultaneous exchange for transcends the private interests of the parties directly involved in
something of value. the transaction and touches the interests of the community at large.
The mischief it creates is not only a wrong to the payee or holder,
but also an injury to the public. In sum, we find the enactment of
B.P. 22 a valid exercise of the police power and is not repugnant to
the constitutional inhibition against imprisonment for debt.
DECS v. SAN Flunking NMAT 5 Police power is validly The private respondent is a W/N a person NO. The proper exercise of the police power requires the
DIEGO times exercised if (a) the interests of graduate of the University of the who failed NMAT 3 concurrence of a lawful subject and a lawful method. The subject of
GR No. 89572; the public generally, as East with a degree of Bachelor of times is entitled to the challenged regulation is certainly within the ambit of the police
December 21, distinguished from those of a Science in Zoology. The petitioner take it again power. It is the right and indeed the responsibility of the State to
1989 particular class, require the claims that he took the NMAT insure that the medical profession is not infiltrated by incompetents
interference of the State, and three times and flunked it as many to whom patients may unwarily entrust their lives and health. The
(b) the means employed are times. When he applied to take it method employed by the challenged regulation is not irrelevant to
reasonably necessary to the again, the petitioner rejected his the purpose of the law nor is it arbitrary or oppressive. The three-
attainment of the object application on the basis of the flunk rule is intended to insulate the medical schools and ultimately
sought to be accomplished and aforesaid rule. He then went to the the medical profession from the intrusion of those not qualified to
not unduly oppressive upon Regional Trial Court of Valenzuela, be doctors. While every person is entitled to aspire to be a doctor,
individuals. Metro Manila, to compel his he does not have a constitutional right to be a doctor. This is true of
admission to the test. any other calling in which the public interest is involved; and the
closer the link, the longer the bridge to one's ambition. The State
has the responsibility to harness its human resources and to see to
it that they are not dissipated or, no less worse, not used at all.
These resources must be applied in a manner that will best promote
the common good while also giving the individual a sense of
satisfaction.

The contention that the challenged rule violates the equal


protection clause is not well- taken. A law does not have to operate
with equal force on all persons or things to be conformable to
Article III, Section 1 of the Constitution. There can be no question
that a substantial distinction exists between medical students and
other students who are not subjected to the NMAT and the three-
flunk rule. The medical profession directly affects the very lives of
the people, unlike other careers which, for this reason, do not
require more vigilant regulation. There would be unequal protection
if some applicants who have passed the tests are admitted and
others who have also qualified are denied entrance. In other words,
what the equal protection requires is equality among equals. The
Court feels that it is not enough to simply invoke the right to quality
education as a guarantee of the Constitution: one must show that
he is entitled to it because of his preparation and promise. The
private respondent has failed the NMAT five times. While his
persistence is noteworthy, to say the least, it is certainly misplaced,
like a hopeless love.
YNOT v. IAC Prohibition of The police power is simply In 1980 President Marcos W/N IAC erred in YES. In the case at bar, E.O. 626-A has the same lawful subject as the
GR No. 74457; transportation of defined as the power inherent amended Executive Order No. 626- upholding the original executive order but NOT the same lawful method. The
March 20, 1987 carabeefs and in the State to A which orders that no carabao validity of EO626A; reasonable connection between the means employed and the
carabaos regulate liberty and property and carabeef shall be transported unconstitutionality purpose sought to be achieved by the questioned measure is
for the promotion of the from one province to another; of EO 626A missing. The challenged measure is an invalid exercise of the police
general welfare. As long as the such violation shall be subject to power because the method employed to conserve the carabaos is
activity or the property has confiscation and forfeiture by the not reasonably necessary to the purpose of the law and, worse, is
some relevance to the government. unduly oppressive. 
public welfare, its regulation
under the police power is not On January 13, 1984, Petitioner’s 6 Due process is violated because the owner of the property
only proper but necessary. carabaos were confiscated by the confiscated is denied the right to be heard in his defense and is
police station commander of immediately condemned and punished. The conferment on the
Lawful means – the means Barotac Nuevo, Iloilo for having administrative authorities of the power to adjudge the guilt of the
employed must be reasonable been transported from Masbate to supposed offender is a clear encroachment on judicial functions and
and must conform to the Iloilo in violation of EO 626-A. He militates against the doctrine of separation of powers. There is,
safeguards guaranteed by the issued a writ for replevin, finally, also an invalid delegation of legislative powers to the officers
Bill of Rights challenging the constitutionality of mentioned therein who are granted unlimited discretion in the
said EO. The trial court sustained distribution of the properties arbitrarily taken.
Lawful subject – the the confiscation of the animals and
activity/property sought to be declined to rule on the validity of
regulated affects the public the law on the ground that it
welfare lacked authority to do so. Its
decision was affirmed by the IAC.
Hence, this petition for review
filed by Petitioner.
QC v. ERICTA Private memorial Police power is usually Sec. 9. At least 6% of the total area W/N Section 9 of NO. It is usually exerted in order to merely regulate the use and
GR No. L-34915; cemetery exercised in the form of mere of the memorial park cemetery the ordinance in enjoyment of property of the owner. If he is deprived of his
June 24, 1983 regulation or restriction in the shall be set aside for charity burial question a valid property outright, it is not taken for public use but rather to destroy
use of liberty or property for of deceased persons who are exercise of the in order to promote the general welfare. In police power, the owner
the promotion of the general paupers and have been residents police power does not recover from the government for injury sustained in
welfare. It does not involve the of Quezon City for at least 5 years consequence thereof.
taking or confiscation of prior to their death, to be
property with the exception of determined by competent City There is no reasonable relation between the setting aside of at least
a few cases where there is a Authorities.” For several years, the six (6) percent of the total area of all private cemeteries for charity
necessity to confiscate private aforequoted section of the burial grounds of deceased paupers and the promotion of health,
property in order to destroy it Ordinance was not enforced by morals. good order, safety, or the general welfare of the people.
for the purpose of protecting city authorities but 7 years after The ordinance is actually a taking without compensation of a certain
the peace and order and of the enactment of the ordinance, area from a private cemetery to benefit paupers who are charges of
promoting the general welfare the Quezon City Council passed the municipal corporation. Instead of building or maintaining a
the a resolution requesting the public cemetery for this purpose, the city passes the burden to
City Engineer to stop any further private cemeteries.
selling and/or transaction of
memorial park lots in Quezon City
where the owners thereof have
failed to donate the required 6%
space intended for paupers burial.
Respondent Himlayang Pilipino,
Inc. contends that the taking or
confiscation of property is obvious
because the questioned ordinance
permanently restricts the use of
the property such that it cannot be
used for any reasonable purpose
and deprives the owner of all
beneficial use of his property.
ASSOC. OF SMALL CARP; Retention It must be noted that like The cases involved have been W/N RA 6657, PD YES. The cases before us present no knotty complication insofar as
LANDOWNERS v. limits for small taxation, the power of eminent consolidated because they 57, Proc. 31 & Eos the question of compensable taking is concerned. To the extent that
SEC. OF landowners domain could be used as an concern common legal questions, 228 & 229 the measures under challenge merely prescribe retention limits for
AGRARIAN implement of police power of including serious challenges to the considered as valid landowners, there is an exercise of the police power for the
REFORM the state. The expressed constitutionality of RA 6657 and constitutional regulation of private property in accordance with the Constitution.
GR No. 78742; objective of the law was the otherwise known as the But where, to carry out such regulation, it becomes necessary to
July 14, 1989 promotion of the welfare of Comprehensive Agrarian Reform deprive such owners of whatever lands they may own in excess of
the farmers, which came Law of 1988 and other the maximum area allowed, there is definitely a taking under the
clearly under the police power supplementary to RA 6657 such as power of eminent domain for which payment of just compensation
of the state. PD 27 (providing for the is imperative. The taking contemplated is not a mere limitation of
compulsory acquisition of private the use of the land. What is required is the surrender of the title to
lands for distribution among and the physical possession of the said excess and all beneficial
tenant-farmers and to specify rights accruing to the owner in favor of the farmer-beneficiary. This
maximum land ownership in favor is definitely an exercise not of the police power but of the power of
of the beneficiaries of PD 27) Pres. eminent domain.
Proc. 131 (instituting CARP) and EO
It bears repeating that the measures challenged in these petitions
229 (providing the mechanics of
contemplate more than a mere regulation of the use of private
implementation of CARP) RA 6657
lands under the police power. We deal here with an actual taking of
is the most recent legislation,
private agricultural lands that has dispossessed the owners of their
signed into law by Pres. Aquino on
property and deprived them of all its beneficial use and enjoyment,
June 10, 1988.
to entitle them to the just compensation mandated by the
Constitution.
The expropriation before us affects all private agricultural lands
whenever found and of whatever kind as long as they are in excess
of the maximum retention limits allowed their owners. This kind of
expropriation is intended for the benefit not only of a particular
community or of a small segment of the population but of the entire
Filipino nation, from all levels of our society, from the impoverished
farmer to the land-glutted owner. Its purpose does not cover only
the whole territory of this country but goes beyond in time to the
foreseeable future, which it hopes to secure and edify with the
vision and the sacrifice of the present generation of Filipinos
MANILA Senior Citizen Act  While the Constitution On April 23, 1992, RA 7432 or the W/N Section 4 of NO. The 20% discount is intended to improve the welfare of senior
MEMORIAL v. protects property rights, Seniors Citizens Act was passed RA No. 9257 and citizens who, at their age, are less likely to be gainfully employed,
SEC. OF DSWD petitioners must accept the into law granting 20% discounts its Implementing more prone to illnesses and other disabilities, and, thus, in need of
GR No. 175356; realities of business and the from all establishments relative to Rules and subsidy in purchasing basic commodities. It may not be amiss to
December 3, 2013 State, in the exercise of police utilization of transportation Regulations, mention also that the discount serves to honor senior citizens who
power, can intervene in the services, hotels and similar lodging insofar as they presumably spent the productive years of their lives on contributing
operations of a business which establishments, restaurants and provide that the to the development and progress of the nation. 
may result in an impairment of recreation centers and purchase of 20% discount to
property rights in the process. medicine anywhere in the country. Senior Citizens The impact or effect of a regulation, such as the one under
Provided, that private may be claimed as consideration, must, thus, be determined on a case-to-case basis.
establishments may claim the cost tax deduction by Whether that line between permissible regulation under police
as tax credit. the private power and "taking" under eminent domain has been crossed must,
establishments are under the specific circumstances of this case, be subject to proof
invalid and
On February 26, 2004, RA 9257 or unconstitutional. and the one assailing the constitutionality of the regulation carries
the Expanded Senior Citizens Act the heavy burden of proving that the measure is unreasonable,
amended certain provisions of RA oppressive or confiscatory.
7432, granting 20% discount to
qualified senior citizens and the In the case at bar, evidence is indispensable before a determination
establishments may claim the of a constitutional violation can be made because of the following
discounts granted as tax deduction reasons. First, the assailed law, by imposing the senior citizen
based on the net cost of the goods discount, does not take any of the properties used by a business
sold or services rendered: establishment like, say, the land on which a manufacturing plant is
Provided, That the cost of the constructed or the equipment being used to produce goods or
discount shall be allowed as services. Second, rather than taking specific properties of a business
deduction from gross income for establishment, the senior citizen discount law merely regulates the
the same taxable year that the prices of the goods or services being sold to senior citizens by
discount is granted. mandating a 20% discount.

Petitioners Manila Memorial Park,


Note that the law does not impose at what specific price the
Inc. and La Funeraria Paz-Sucat,
product shall be sold, only that a 20% discount shall be given to
Inc., domestic corporations
senior citizens based on the price set by the business establishment.
engaged in the business of
A business establishment is, thus, free to adjust the prices of the
providing funeral and burial
goods or services it provides to the general public.
services, feeling aggrieved by the
tax deduction scheme, petitioners
prayed that Section 4 of RA 9257
and the IRR issued by DSWD and
the DOF be declared
unconstitutional insofar as these
allow business establishments to
claim the 20% discount given to
senior citizens as a tax deduction;
that the DSWD and the DOF be
prohibited from enforcing the
same; and that the tax credit
treatment of the 20% discount
under the former Section 4 (a) of
RA 7432 be reinstated.
PASCUAL v. SEC. Pasig feeder road It is a general rule that the On August 31, 1954, petitioner W/N the incidental NO. . It is the essential character of the direct object of the
OF PUBLIC terminals in legislature is without power to Wenceslao Pascual, as Provincial gains by the public expenditure which must determine its validity as justifying a tax,
WORKS Antonio appropriate public revenue for Governor of Rizal, instituted this be considered and not the magnitude of the interest to be affected nor the degree
GR No. L-10405; Subdivision anything but a public purpose action for declaratory relief, with "public purpose" to which the general advantage of the community, and thus the
December 29, owned by Zulueta injunction upon the ground that for the purpose of public welfare, may be ultimately benefited by their promotion.
1960 Republic Act No. 920, entitled An justifying an Incidental to the public or to the state, which results from the
Act Appropriating Funds for Public expenditure of the promotion of private interest and the prosperity of private
Works" an item of P85,000.00, "for government enterprises or business, does not justify their aid by the use public
the construction, reconstruction, money. The test of the constitutionality of a statute requiring the
repair, extension and use of public funds is whether the statute is designed to promote
improvement" of "Pasig feeder the public interest, as opposed to the furtherance of the advantage
road terminals; that, at the time of of individuals, although each advantage to individuals might
the passage and approval of said incidentally serve the public.
Act, the aforementioned feeder
roads were "nothing but projected
and planned subdivision roads, not
yet constructed, . . . within the
Antonio Subdivision . . . situated at
. . . Pasig, Rizal which projected
feeder roads "do not connect any
government property or any
important premises to the main
highway"; that the
aforementioned Antonio
Subdivision (as well as the lands on
which said feeder roads were to be
constructed) were private
respondent Jose C. Zulueta, who,
at the time of the passage and
approval of said Act, was a
member of the Senate of the
Philippines; that on May 29, 1953,
respondent Zulueta, addressed a
letter to the Municipal Council of
Pasig, Rizal, offering to donate said
projected feeder roads to the
municipality of Pasig, Rizal; The
respondents' contention is that
there is public purpose because
people living in the subdivision will
directly be benefitted from the
construction of the roads, and the
government also gains from the
donation of the land supposed to
be occupied by the streets, made
by its owner to the government. 

PUNSALAN v. Municipal There is double taxation where 2 lawyers, a medical practitioner, a W/N this ruling is NO. Plaintiffs' complaint is that while the law has authorized the City
MUNICIPAL Occupation Tax; one tax is imposed by the state public accountant, a dental doctor. correct, for though of Manila to impose the said tax, it has withheld that authority from
BOARD OF Double Taxation and the other is imposed by The ordinance in question, which the decision is other chartered cities, not to mention municipalities. We do not
MANILA the city, it being widely was approved by the municipal silent on the think it is for the courts to judge what particular cities or
GR No. L-4817; recognized that there is board of the City of Manila on July refund of taxes municipalities should be empowered to impose occupation taxes
May 26, 1954 nothing inherently obnoxious 25, 1950, imposes a municipal paid plaintiffs in addition to those imposed by the National Government. That
in the requirement that license occupation tax on persons make no matter is peculiarly within the domain of the political departments
fees or taxes be enacted with exercising various professions in assignment of and the courts would do well not to encroach upon it. Moreover,
respect to the same the city and penalizes non- error on this point. as the seat of the National Government and with a population and
occupation, calling or activity payment of the tax. The ordinance volume of trade many times that of any other Philippine city or
by both the state and the was enacted pursuant to municipality, Manila, no doubt, offers a more lucrative field for the
political subdivisions thereof. paragraph (1) of section 18 of the practice of the professions, so that it is but fair that the
Revised Charter of the City of professionals in Manila be made to pay a higher occupation tax than
Manila which empowers the their brethren in the provinces.
Municipal Board of said city to
impose a municipal occupation Plaintiffs brand the ordinance unjust and oppressive because they
tax, not to exceed P50 per annum, say that it creates discrimination within a class in that while
on persons engaged in the various professionals with offices in Manila have to pay the tax, outsiders
professions. who have no offices in the city but practice their profession therein
Having already paid their are not subject to the tax. Plaintiffs make a distinction that is not
occupation tax under section 201 found in the ordinance. The ordinance imposes the tax upon every
of the National Internal Revenue person "exercising" or "pursuing" — in the City of Manila naturally
Code, plaintiffs, upon being — any one of the occupations named, but does not say that such
required to pay the additional tax person must have his office in Manila. What constitutes exercise or
prescribed in the ordinance, paid pursuit of a profession in the city is a matter of judicial
the same under protest and then determination
brought the present suit for the
purpose already stated.
The lower court upheld the validity
of the provision of law authorizing
the enactment of the ordinance
but declared the ordinance itself
illegal and void on the ground that
the penalty therein provided for
non-payment of the tax was not
legally authorized.
LLADOC v. Donee’s gift tax A gift tax is not a property tax, On March 3, 1958, the donor M.B. W/N petitioner NO. The head of the diocese and not the parish priest is the real
COMMISSIONER but an excise tax imposed on Estate, Inc., filed the donor's gift should be liable party in interest in the imposition of a donee's tax on property
ON INTERNAL the transfer of property by way tax return. Under date of April 29, for the assessed donated to the church for religious purposes.
REVENUE of gift, the imposition of which 1960, the respondent donee's gift tax on
GR No. L-19201; on property used exclusively Commissioner of Internal Revenue the P10,000.00 Section 22(3), Art. VI of the Constitution of the Philippines, exempts
June 16, 1965 for religious purposes, do not issued as assessment for donee's donated for the from taxation cemeteries, churches and personages or convents,
constitute an impairment of gift tax against the Catholic Parish construction of the appurtenant thereto, and all lands, buildings, and improvements
the Constitution. of Victorias, Negros Occidental, of Victorias Parish used exclusively for religious purposes. The exemption is only from
which petitioner was the priest. Church the payment of taxes assessed on such properties enumerated, as
The tax amounted to P1,370.00 property taxes, as contra-distinguished from excise taxes. In the
including surcharges, interest of present case, what the Collector assessed was a donee's gift tax; the
1% monthly from May 15, 1958 to assessment was not on the properties themselves. It did not rest
June 15, 1960, and the upon general ownership; it was an excise upon the use made of the
compromise for the late filing of properties, upon the exercise of the privilege of receiving the
the return. properties. Manifestly, gift tax is not within the exempting
provisions of the section just mentioned.
In the petition for Review, the Rev.
Fr. Casimiro Lladoc, claimed
among others, that at the time of
the donation, he was not the
parish priest in Victorias; that
there is no legal entity or juridical
person known as the "Catholic
Parish Priest of Victorias," and
therefore, he should not be liable
for the donee's gift tax. It was also
asserted that the assessment of
the gift tax, even against the
Roman Catholic Church, would not
be valid, for such would be a clear
violation of the provisions of the
Constitution.
ABRA VALLEY School property Tax exemptions exclusively for On June 8, 1972 the properties of W/N the lot and NO. The exemption in favor of property used exclusively for
COLLEGE v. used for charitable or educational the Abra Valley Junior College, Inc. building in charitable or educational purposes is 'not limited to property
AQUINO residential purposes only was sold at public auction for the question are used actually indispensable' therefor but extends to facilities which are
GR No. L-39086; purpose by the satisfaction of the unpaid real exclusively for incidental to and reasonably necessary for the accomplishment of
June 15, 1988 Director of property taxes thereon and the educational said purposes, such as in the case of hospitals, 'a school for training
School same was sold to defendant purposes. nurses, a nurses' home, property used to provide housing facilities
Paterno Millare who offered the for interns, resident doctors, superintendents, and other members
highest bid of P6,000.00 and a W/N the building of the hospital staff, and recreational facilities for student nurses,
Certificate of Sale in his favor was in question is interns, and residents' such as 'athletic fields' including 'a farm used
issued by the defendant Municipal subject to tax for the inmates of the institution.'"
Treasurer. exemption as
enumerated in the It must be stressed however, that while this Court allows a more
The trial court among others, Constitution. liberal and non- restrictive interpretation of the phrase "exclusively
found the following: (a) that the used for educational purposes" as provided for in Article VI, Section
school is recognized by the 22, paragraph 3 of the 1935 Philippine Constitution, reasonable
government and is offering emphasis has always been made that exemption extends to facilities
Primary, High School and College which are incidental to and reasonably necessary for the
Courses, and has a school accomplishment of the main purposes. Otherwise stated, the use of
population of more than one the school building or lot for commercial purposes is neither
thousand students all in all; (b) contemplated by law, nor by jurisprudence. Thus, while the use of
that it is located right in the heart the second floor of the main building in the case at bar for
of the town of Bangued, a few residential purposes of the Director and his family, may find
meters from the plaza and about justification under the concept of incidental use, which is
120 meters from the Court of First complimentary to the main or primary purpose — educational, the
Instance building; (c) that the lease of the first floor thereof to the Northern Marketing
elementary pupils are housed in a Corporation cannot by any stretch of the imagination be considered
two-storey building across the incidental to the purpose of education.
street; (d) that the high school and
college students are housed in the Under the 1935 Constitution, the trial court correctly arrived at the
main building; (e) that the Director conclusion that the school building as well as the lot where it is
with his family is in the second built, should be taxed, not because the second floor of the same is
floor of the main building; and (f) being used by the Director and his family for residential purposes,
that the annual gross income of but because the first floor thereof is being used for commercial
the school reaches more than one purposes. However, since only a portion is used for purposes of
hundred thousand pesos. commerce, it is only fair that half of the assessed tax be returned to
the school involved.
Petitioner contends that the
primary use of the lot and building
for educational purposes, and not
the incidental use thereof,
determines the exemption from
property taxes under Section 22
(3), Article VI of the 1935
Constitution. Hence, the seizure
and sale of subject college lot and
building, which are contrary
thereto as well as to the provision
of Commonwealth Act No. 470,
otherwise known as the
Assessment Law, are without legal
basis and therefore void.

On the other hand, private


respondents maintain that the
college lot and building in question
which were subjected to seizure
and sale to answer for the unpaid
tax are used: (1) for the
educational purposes of the
college; (2) as the permanent
residence of the President and
Director thereof, Mr. Pedro V.
Borgonia, and his family including
the in-laws and grandchildren; and
(3) for commercial purposes
because the ground floor of the
college building is being used and
rented by a commercial
establishment, the Northern
Marketing Corporation
SISON JR. v. Imposition of The power to tax, an inherent Petitioners challenged the W/N the assailed NO. The taxing power has the authority to make reasonable and
ANCHETA higher rates of prerogative, has to be availed constitutionality of Section 1 of provision violates natural classifications for purposes of taxation. Where “the
GR No. L-59431; tax of to assure the performance Batas Pambansa Blg. 135. It the equal differentiation” complained of “conforms to the practical dictates of
July 25, 1984 of vital state functions. It is the amended Section 21 of protection and justice and equity” it “is not discriminatory within the meaning of
source of the bulk of public the National Internal due process clause this clause and is therefore uniform.” There is quite a similarity then
funds. Revenue Code of 1977. Petitioner of the Constitution to the standard of equal protection for all that is required is that the
as taxpayer alleged that "he would while also violating tax “applies equally to all persons, firms and corporations placed in
Equality and uniformity in be unduly discriminated against by the rule that taxes similar situation.
taxation means that all taxable the imposition of higher rates of must be uniform On equal protection: it is suffices that the laws operate equally and
articles or kinds of property of tax upon his income arising and equitable. uniformly on all persons under similar circumstances, both in the
the same class shall be taxed at from the exercise of his profession privileges conferred and the liabilities imposed.
the same rate. The taxing vis-a-vis those which are imposed On due process: it is undoubted that it may be invoked where a
power has the authority to upon fixed income or taxing statute is so arbitrary that it finds no support in the
make reasonable and natural salaried individual taxpayers." He Constitution. An obvious example is where it can be shown to
classifications for purposes of characterizes the above section as amount to the confiscation of property from abuse of power.
taxation. arbitrary amounting to class Petitioner alleges arbitrariness but his mere allegation does not
legislation, oppressive and suffice and there must be a factual foundation of such
capricious in character. For unconstitutional taint. Petitioner alleges arbitrariness. A mere
petitioner, therefore, there is a allegation does not suffice. There must be a factual foundation of
transgression of both the equal such unconstitutional taint.
protection and due process clauses
of the Constitution as well as of
the rule requiring uniformity in
taxation. The OSG prayed for
dismissal of the petition due to
lack of merit.
TOLENTINO v. e-VAT; Regressive Regressive is not a negative The value-added tax (VAT) is levied W/N eVAT is NO. The Constitution does not really prohibit the imposition of
SEC. OF FINANCE standard for courts to enforce. on the sale, barter or exchange of unconstitutional regressive taxes. What it simply provides is that Congress shall
GR No. 115455; What Congress is required by goods and properties as well as on evolve a progressive system of taxation. The Constitutional
August 25, 1994 the Constitution to do is to the sale or exchange of services. It provision should be construed to mean simply that “direct taxes are
“evolve a progressive system is equivalent to 10% of the gross to be preferred and indirect taxes, as much as possible, should be
of taxation.” This is a directive selling price or gross value in minimized.” (E. Fernando, Constitution of the Philippines) Indeed,
to Congress, just like the money of goods or properties sold, the mandate to Congress is not to prescribe, but to evolve a
directive to it to give priority to bartered or exchanged or of the progressive tax system. This is a mere directive upon Congress, not
the enactment of laws for the gross receipts from the sale or a justiciable right or a legally enforceable one. We cannot avoid
enhancement of human dignity exchange of services. regressive taxes but only minimize them.
and the reduction of social,
economic and political Republic Act No. 7716 seeks to
inequalities [Art. XIII, Sec. 1] or widen the tax base of the existing
for the promotion to the right VAT system and enhance its
to “quality education” [Art. administration by amending the
XIV, Sec.1] These provisions National Internal Revenue Code.
are put in the Constitution as
moral incentives to legislation, It was challenged for alleged
not judicially enforceable constitutional infirmities (defects),
rights. among others: that the law did not
originate exclusively in the House
Lifeblood Doctrine - taxes are of Representative as required by
what we pay for civilized Section 24, Article VI – they
society. Without taxes, the contended that to be considered
government would be as having originated in the HOR, it
paralyzed for lack of the should retain the essence of the
motive power to activate and House Bill.
operate it. Hence, despite the
natural reluctance to surrender
part of one’s hard-earned
income to the taxing
authorities, every person who
is able to must contribute his
share in the burden of running
the government. The
government, for its part, is
expected to respond in the
form of tangible and intangible
benefits intended to improve
the lives of the people and
enhance their material and
moral values.”
PBMEO v. PBM The primacy of human rights Sometime in 1969, petitioner
over property rights is decided to stage a mass
recognized. In the hierarchy of demonstration in Malacañang in
civil liberties, the rights of free protest against alleged abuses of
expression and of assembly the Pasig Police. Respondent
occupy a preferred position as Company, however requested
they are essential to the petitioner that the first-shift
preservation and vitality of our workers should not participate in
civil and political institutions. the strike for it will unduly
prejudice the normal operation of
“The superiority of these the company. Despite the warning,
freedoms over property rights all the workers including those
is underscored by the fact that who were in first-shift still
a mere reasonable or rational participated in the rally. Prior to
relation between the means that, respondent company
employed by the law and its informed that workers who belong
object or purpose – that the in the first-shift, who were without
law is neither arbitrary nor previous leave of absence
discriminatory nor oppressive- approved by the company, who
would suffice to validate a law shall participate in the rally shall
which restricts or impairs be dismissed for it is a clear
property rights. On the other violation of the existing CBA and is
hand, a constitutional or valid tantamount to an illegal strike.
infringement of human rights Respondent company then filed a
requires a more stringent charge against petitioners and
criterion, namely existence of a later dismissed some of its
grave and immediate danger of employees.
a substantive evil which the
State has the right to prevent.”
SIMON v. CHR Demolition of The term “civil rights,” has A “Demolition Notice” was signed W/N the CHR’s YES. In the deliberations of the Constitutional Commission, it
GR No. 100150; shanties along been defined as referring: by Carlos Quimpo. In said Notice, jurisdiction is apparent that the delegates envisioned a CHR that would focus its
January 5, 1994 North EDSA “to those rights that belong to the respondents were given a confined only to attention to the more severe cases of human rights violations. One
every citizen of the state or grace period of 3 days within the investigation of the delegates, for instance, mentioned such areas as the “(1)
country, or, in a wider sense, which to vacate the premises of of violations of protection of rights of political detainees, (2) treatment of prisoner
to all its inhabitants, and are North EDSA. Prior to their receipt civil and political and the prevention of tortures, (3) fair and public trials, (4) cases of
not connected with the of the demolition notice, the PRs rights disappearances, (5) salvaging and hamletting, and (6) other crimes
organization or administration were informed by petitioner committed against the religious.” In any event, the delegates did not
of government. They include Quimpo that their stalls should be apparently take comfort in peremptorily making a conclusive
the rights of property, removed to give way to the delineation of the CHR’s scope of investigatorial jurisdiction. They
marriage, equal protection of “People’s Park.” have thus seen it fit to resolve, instead, that “Congress may provide
the laws, freedom of contract, On July 12 1990, the group, led by for other cases of violations of human rights that should fall within
etc. Or, as otherwise defined their President Roque Ferno, filed the authority of the Commission, taking into account its
civil rights are rights a letter-complaint with the CHR recommendation.”
appertaining to a person by against the petitioners, asking the
virtue of his citizenship in a late CHR Chairman Mary In the particular case at hand, there is no cavil that what are sought
state or community. It may Concepcion Bautista for a letter to be demolished are the stalls, sarisari stores and carinderia, as
also refer, in its general sense, addressed to then Mayor Brigido well as temporary shanties, erected by PRson a land which is
to rights capable of being Simon, Jr., of QC to stop the planned to be developed into a “People’s Park.” More than that, the
enforced or redressed in a civil demolition. , the CHR issued an land adjoins the North EDSA of QC which, this Court can take judicial
action.” order, directing the petitioners “to notice of, is a busy national highway. The consequent danger to life
Political rights, on the other desist from demolishing the stalls and limb cannot thus to be likewise simply ignored. It is indeed
hand, are said to refer to the and shanties at North Edsa paradoxical that a right which is claimed to have been violated is
right to participate, directly or pending resolution of the one that cannot, in the first place, even be invoked, if it is not, in
indirectly, in the establishment vendors/squatters’ complaint fact, extant. Be that as it may, looking at the standards discoursed
or administration of before the Commission” visavis the circumstances obtaining in this instance, we are not
government, the right of On Sept. 10, 1990, a motion to prepared to conclude that the order for the demolition of the stalls,
suffrage, the right to hold dismiss (MD) filed by the sarisari stores and carinderia of the PRs can fall within the
public office, the right of petitioners before the CHR compartment of “human rights violations involving civil and political
petition and, in general, the questioned CHR’s jurisdiction. It rights” intended by the Constitution.
right appurtenant to was stated that the CHR’s
citizenship. authority should be understood as
being confined only to the
investigation of violations of civil
and political rights, and that “the
rights allegedly violated not such
rights but privilege to engage in
business.”
BANCO ESPANOL Publication The jurisdiction being once Engracio Palanca Tanquinyeng y W/N due process YES. the observations of the Court led to the conclusions: (1) that
v. PALANCA established, all that due Limquingco mortgaged various was observed by the failure of the clerk to send the notice to the defendant by mail
process of law thereafter parcels of real property in Manila the court did not destroy the jurisdiction of the court and (2) that such
requires is an opportunity for to El Banco Espanol-Filipino. irregularity did not infringe the requirement of due process of law.
the defendant to be heard; and Afterwards, Engracio returned to Notice was given by publication in a newspaper and this is the only
as publication was duly made China and there he died on form of notice which the law unconditionally requires. This is all that
in the newspaper, it would January 29, 1810 without was absolutely necessary to sustain the proceedings. Property is
seem highly unreasonable to returning again to the Philippines. always assumed to be in the possession of its owner, in person or by
hold that failure to mail the The mortgagor then instituted agent; and he may be safely held, under certain conditions, to be
notice was fatal. foreclosure proceeding but since affected with knowledge that proceedings have been instituted for
defendant is a non-resident, it was its condemnation and sale.
necessary to give notice by
publication. The court either has jurisdiction or it has not; and if the requirement
The Clerk of Court was also as to the mailing of notice should be considered as a step
directed to send copy of the antecedent to the acquiring of jurisdiction, there could be no escape
summons to the defendant’s last from the conclusion that the failure to take that step was fatal to
known address, which is in Amoy, the validity of the judgment. In the application of the idea of due
China. It is not shown whether the process of law, on the other hand, it is clearly unnecessary to be so
Clerk complied with this rigorous
requirement. Nevertheless, after
publication in a newspaper of the
City of Manila, the cause
proceeded and judgment by
default was rendered.
The decision was likewise
published and afterwards sale by
public auction was held with the
bank as the highest bidder. On
August 7, 1908, this sale was
confirmed by the court. However,
about seven years after the
confirmation of this sale, a motion
was made by Vicente Palanca, as
administrator of the estate of the
original defendant, wherein the
applicant requested the court to
set aside the order of default and
the judgment, and to vacate all the
proceedings subsequent thereto.
The basis of this application was
that the order of default and the
judgment rendered thereon were
void because the court had never
acquired jurisdiction over the
defendant or over the subject of
the action.

GALVEZ v. CA Murder case It is a general rule that a nolle On November 12, 1993, Whether the YES. Petitioners aver that they were requesting for the suspension
GR No. 114046; prosequi or dismissal entered petitioners Honorato Galvez, the arraignment of the arraignment as they wanted to have a final copy of the order
October 24, 1994  before the accused is placed incumbent Mayor of San Ildefonso, proceeding held of January 24, 1994 which was merely read in open court, and to
on trial and before he is called Bulacan, and one Godofredo Diego on January 24, take the necessary steps to question the same by way of a motion
on to plead is not equivalent to were charged in three separate 1994 was valid. for reconsideration or an appeal.
an acquittal, and does not bar informations with homicide and
a subsequent prosecution for two counts of frustrated homicide In criminal cases, it is the duty of the accused, in addition to the
the same offense. It is not a for allegedly shooting to death other pleas authorized by law, to plead whether he is guilty or not
final disposition of the case. Alvin Calma Vinculado and of the crime charged. In that way and in that way only can an issue
Rather, it partakes of the seriously wounding Levi Calma be created upon which the trial shall proceed. Section 1(c) of Rule
nature of a nonsuit or Vinculado and Miguel Reyes 116 is quite explicit that where the accused refuses to plead, a plea
discontinuance in a civil suit Vinculado, Jr. of not guilty shall be entered for him. Hence, under such mandatory
and leaves the matter in the language, if the accused refuses to plead, the court must enter a
same condition in which it was On November 15, 1993, Bulacan plea of not guilty. The words are so plain and unambiguous that no
before the commencement of Provincial Prosecutor Liberato L. construction is necessary. It actually calls for a literal application
the prosecution. Reyes filed a Motion to Defer thereof. Any explanation or defense which petitioners would want
Arraignment and Subsequent to invoke can be properly raised during the trial, but they cannot
A dismissal is different from an Proceedings to enable him "to refuse to enter their plea. Nonetheless, the alleged defect in their
acquittal. An order of dismissal review the evidence on record and arraignment on January 24, 1994 is deemed to have been cured
which is actually an acquittal is determine once more the proper when they were again arraigned on February 18, 1994 with the
immediately final and cannot crimes chargeable against the assistance of counsel de  oficio, and the information was read to
be reconsidered. Furthermore, accused," which was granted by them in the vernacular.
an acquittal is always based on Judge Villajuan in an order dated
the merits, that is, the November 16, 1993. Thereafter,
defendant is acquitted because pursuant to Department Order No.
the evidence does not show 369 of the Department of Justice,
that defendant's guilt is respondent Prosecutor Dennis M.
beyond reasonable doubt; but Villa-Ignacio was designated Acting
a dismissal does not decide the Provincial Prosecutor of Bulacan
case on the merits or that the and was instructed to conduct a
defendant is not guilty. re-investigation of the aforesaid
Dismissals terminate the criminal cases filed against herein
proceedings, either because petitioners.
the court is not a court of
competent jurisdiction, or the On December 15, 1993, before
evidence does not show that petitioners could be arraigned in
the offense was committed Criminal Cases, respondent
within the territorial prosecutor filed an Ex Parte
jurisdiction of the court, or the Motion to Withdraw Informations
complaint or information is not in said cases. This motion was
valid or sufficient in form and granted by Judge Villajuan also on
substance.  For dismissal to be December 15, 1993 and the cases
a bar under double jeopardy, it were considered withdrawn from
must have the effect of the docket of the court. On
acquittal. the same day, Prosecutor Villa-
Ignacio filed four new informations
against herein petitioners for
murder, two counts of frustrated
murder, and violation of
Presidential Decree No. 1866 for
illegal possession of
firearms which were subsequently
raffled to the sala of Judge Victoria
Pornillos
On December 27, 1993, the
scheduled arraignment before
Judge Pornillos was reset due to
the absence of respondent
prosecutor. On even date,
petitioners filed before Judge
Villajuan a Motion for
Reconsideration of his order of
December 15, 1993 which granted
the motion to withdraw the
original informations.
In the meantime, and prior to the
arraignment of herein petitioners
before Judge Pornillos, an order
was issued on January 20, 1994 by
Judge Villajuan granting the
motion for reconsideration filed by
petitioners, ordering the
reinstatement of Criminal Cases,
and setting the arraignment of the
accused therein for February 8,
1994.On said date, however, the
arraignment was suspended and,
in the meanwhile, petitioners filed
a petition for certiorari, prohibition
and mandamus with respondent
Court of Appeals assailing the
order dated January 24, 1994
issued by Judge Pornillos which
denied petitioners' motion to
quash.
STATE Misconduct Matters of judicial notice have The state prosecutors who are W/N respondent YES. To say that a court will take judicial notice of a fact is merely
PROSECUTORS v. three material requisites: members of the DOJ Panel of judge commit another way of saying that the usual form of evidence will be
MURO 1. the matter must be Prosecution filed a complaint grave abuse of dispensed with if knowledge of the fact can be otherwise acquired.
one of common and against respondent Judge Muro on discretion in taking This is because the court assumes that the matter is so notorious
general knowledge; the ground of ignorance of the judicial notice on that it will not be disputed. But judicial notice is not judicial
2. it must be well and law, grave misconduct and the statement of knowledge.
authoritatively violation of the provisions in the the president
settled and not Code of Judicial Conduct. lifting the foreign The mere personal knowledge of the judge is not the judicial
doubtful or The case at bar involves the exchange knowledge of the court, and he is not authorized to make his
uncertain; prosecution of the 11 charges restriction individual knowledge of a fact, not generally or professionally
3. it must be known to against Imelda Marcos in violation published in the known, the basis of his action. Judicial cognizance is taken only of
be within the limits of the Central Bank Foreign newspaper as those matters which are “commonly” known.
of the jurisdiction of Exchange Restriction in the Central basis for
the court. Bank Circular 960. dismissing the case In this case, respondent judge, in the guise of exercising discretion
The principal guide in The respondent judge dismissed all and on the basis of a mere newspaper account which is sometimes
determining what facts may be 11 cases solely on the basis of the even referred to as hearsay evidence twice removed, took judicial
assumed to be judicially known report published from the 2 notice of the supposed lifting of foreign exchange controls, a matter
is that of notoriety. Hence, it newspapers (Inquirer and Daily which was not and cannot be considered of common knowledge or
can be said that judicial notice Globe), which the judge believes of general notoriety. Worse, he took cognizance of an
is limited to facts evidenced by to be reputable and of national administrative regulation which was not yet in force when the order
public records and facts of circulation, that the Pres. of the of dismissal was issued.
general notoriety. Philippines lifted all foreign
exchange restrictions. Jurisprudence dictates that judicial notice cannot be taken of a
The doctrine of judicial notice He further contends that the statute before it becomes effective. The reason is simple. A law
rests on the wisdom and announcement of the President as which is not yet in force and hence, still inexistent, cannot be of
discretion of the courts. The published in the newspaper has common knowledge capable of ready and unquestionable
power to take judicial notice is made such fact a public knowledge demonstration, which is one of the requirements before a court can
to be exercised by courts with that is sufficient for the judge to take judicial notice of a fact.
caution; care must be taken take judicial notice which is
that the requisite notoriety discretionary on his part.
exists; and every reasonable Hence, the complainants contend
doubt on the subject should be that the respondent judge erred in
promptly resolved in the taking judicial notice on matters he
negative. purported to be a public
knowledge based merely on the
account of the newspaper
publication that the Pres. has lifted
the foreign exchange restriction.
It was also an act of inexcusable
ignorant of the law not to accord
due process to the prosecutors
who were already at the stage of
presenting evidence (trial)thereby
depriving the government the
right to be heard.
The judge also exercised grave
abuse of discretion by taking
judicial notice on the published
statement of the President in the
newspaper which is a matter that
has not yet been officially in force
and effect of the law.
CARVAJAL v. CA Inheritance The essence of due process is Petitioner allegedly acquired W/N the NO. In this case, petitioner was afforded an opportunity to present
GR No. 98328, the opportunity to be heard. It portions of the parcel of land by petitioners were witnesses and he did present three. However, petitioner did not
October 9, 1997 is the denial of this opportunity inheritance from his father Felix denied due invoke his right to take the witness stand even when the trial court
that is repugnant to due Carvajal who came to possess the process of law ordered the submission of the parties' memoranda which signified
process. unregistered land in 1938, the termination of the proceedings. Because he acquiesced to a the
continuously, openly, adversely termination of the case, he forfeited his right to take the witness
and peacefully in the concept of an stand. Likewise, we are not persuaded by his allegation that his own
owner up to the time of his death. counsel hardly participated in the proceedings In any event, the
client is generally bound by the acts of his counsel Petitioner has not
The latter court upheld the trial shown at all that his previous counsel had acted in such grossly
court LRC filed before RTC negligent manner as to deprive him of effective representation, or
Antipolo In dismissing petitioner's of due process.
application for registration of title
of a parcel of land in Antipolo City.
The Court recognized respondent
Solid Homes, Inc. as the registered
owner of a parcel of land covered
by TCT No. N-7873, situated in
Antipolo City.

The Court of Appeals affirmed the


dismissal of the application for
registration, and denied the
subsequent motion for
reconsideration.

Petitioner maintains that he was


denied due process when he, as an
applicant in a land registration
case, was not able to take the
witness stand.  According to
petitioner, even his counsel hardly
participated in the proceeding
except to propound clarificatory
questions during the examination
of Engineer Silverio Perez of the
Land Registration Authority
PEREZ v. Live media Freedom of the press v. Rights On 13 March 2001, the Kapisanan The courts recognize the constitutionally embodied freedom of the
ESTRADA coverage of of the accused ng mga Brodkaster ng press and the right to public information. It also approves of media's
JUNE 29, 2001 Estrada’s trial Pilipinas(KBP), an association exalted power to provide the most accurate and comprehensive
Due process guarantees the representing duly franchised and means of conveying the proceedings to the public and in
accused a presumption of authorized television and radio acquainting the public with the judicial process in action;
innocence until the contrary is networks throughout the country, nevertheless, within the courthouse, the overriding consideration is
proved in a trial that is not sent a letter requesting this Court still the paramount right of the accused to due process which must
lifted above its individual to allow live media coverage of the never be allowed to suffer diminution in its constitutional
settings nor made an object of anticipated trial of the plunder and proportions. Justice Clark thusly pronounced, "while a maximum
public's attention and where other criminal cases 􏰄led against freedom must be allowed the press in carrying out the important
the conclusions reached are former President Joseph E. Estrada function of informing the public in a democratic society, its exercise
induced not by any outside before the Sandiganbayan in order must necessarily be subject to the maintenance of absolute fairness
force or influence but only by "to assure the public of full in the judicial process."
evidence and argument given transparency in the proceedings of An accused has a right to a public trial but it is a right that belongs
in open court, where fitting an unprecedented case in our to him, more than anyone else, where his life or liberty can be held
dignity and calm ambiance is history." The request was critically in balance. A public trial aims to ensure that he is fairly
demanded. seconded by Mr. Cesar N. Sarino in dealt with and would not be unjustly condemned and that his rights
his letter of 05 April 2001 to the are not compromised in secret conclaves of long ago. A public trial is
Chief Justice and, still later, by not synonymous with publicized trial; it only implies that the court
Senator Renato Cayetano and doors must be open to those who wish to come, sit in the available
Attorney Ricardo Romulo. seats, conduct themselves with decorum and observe the trial
The foregoing criminal cases process. In the constitutional sense, a courtroom should have
involve the previous acts of the enough facilities for a reasonable number of the public to observe
former highest official of the land, the proceedings, not too small as to render the openness negligible
members of his family, his cohorts and not too large as to distract the trial participants from their
and, therefore, it cannot be over proper functions, who shall then be totally free to report what they
emphasized that the prosecution have observed during the proceedings.
thereof, definitely involves a
matter of public concern and Unlike other government offices, courts do not express the popular
interest, or a matter over which will of the people in any sense which, instead, are tasked to only
the entire citizenry has the right to adjudicate justiciable controversies on the basis of what alone is
know, be informed and made submitted before them. A trial is not a free trade of ideas. Nor is a
aware of. competing market of thoughts the known test of truth in a
There is no gainsaying that the courtroom.
constitutional right of the people
to be informed on matters of
public concern, as in the instant
cases, can best be recognized,
served and satisfied by allowing
the live radio and television
coverage of the concomitant court
proceedings. Moreover, the live
radio and television coverage of
the proceedings will also serve the
dual purpose of ensuring the
desired transparency in the
administration of justice in order
to disabuse the minds of the
supporters of the past regime of
any and all unfounded notions, or
ill-perceived attempts on the part
of the present dispensation, to
'railroad' the instant criminal cases
against the Former President
Joseph Ejercito Estrada
SEPTEMBER 13, This motion was filed by the The Court ruled that considering the significance of the trial before
2001 Secretary of Justice, as one of the the Sandiganbayan of former President Estrada and the importance
petitioners, who argued that there of preserving the records thereof, the Court believes that there
is really no conflict between the should be an audio-visual recording of the proceedings. The
right of the people to public recordings will not be for live or real time broadcast but for
information and the freedom of documentary purposes. Only later will they be available for public
the press, on one hand, and, on showing, after the Sandiganbayan shall have promulgated its
the other, the right of the accused decision in every case to which the recording pertains. The master
to a fair trial. He further film shall be deposited in the National Museum and in the Records
elaborated that if there is a clash Management and Archives Office for historical preservation and
between these rights, it must be exhibition pursuant to law.
resolved in favor of the right of the For the purpose of recording the proceedings, cameras will be
people and the press because the inconspicuously installed in the courtroom and the movement of TV
people, as the repository of crews will be regulated, consistent with the dignity and solemnity of
sovereignty, are entitled to the proceedings. The trial shall be recorded in its entirety, except
information, and live media such portions thereof as the Sandiganbayan may decide should not
coverage is a safeguard against be held public pursuant to Rule 119, §21 of the Revised Rules of
attempts by any party to use the Criminal Procedure. No comment shall be included in the
courts as instruments for the documentary except annotations which may be necessary to explain
pursuit of selfish interests. On the certain scenes which are depicted. The audio-visual recordings shall
other hand, former President be made under the supervision and control of the Sandiganbayan or
Joseph E. Estrada reiterated his its Division as the case may be.
objection to the live TV and radio There are several reasons for such televised recording. First, the
coverage of his trial on the ground hearings are of historic significance. They are an affirmation of our
that its allowance will violate the commitment to the rule that "the King is under no man, but he is
sub judice rule. under God and the law."
The recordings will be useful in preserving the essence of the
proceedings in a way that the cold print cannot quite do because it
cannot capture the sights and sounds of events. They will be
primarily for the use of appellate courts in the event a review of the
proceedings, rulings, or decisions of the Sandiganbayan is sought or
becomes necessary. The accuracy of the transcripts of stenographic
notes taken during the trial can be checked by reference to the
tapes
On the other hand, by delaying the release of the tapes for
broadcast, concerns that those taking part in the proceedings will
be playing to the cameras and will thus be distracted from the
proper performance of their roles — whether as counsel, witnesses,
court personnel, or judges — will be allayed. The possibility that
parallel trials before the bar of justice and the bar of public opinion
may jeopardize, or even prevent, the just determination of the
cases can be minimized. The possibility that judgment will be
rendered by the popular tribunal before the court of justice can
render its own will be avoided. At the same time, concerns about
the regularity and fairness of the trial — which, it may be assumed,
is the concern of those opposed to, as much as of those in favor of,
televised trials — will be addressed since the tapes will not be
released for public showing until after the decision of the cases by
the Sandiganbayan. By delaying the release of the tapes, much of
the problem posed by real time TV and radio broadcast will be
avoided.
ANG TIBAY v. CIR Substantial There are cardinal primary Teodoro Toribio owns and W/N the National YES. In this case the records show that the newly discovered
GR No. 46496; Evidence rights which must be respected operates Ang Tibay, a leather Labor Union, Inc. is evidence or documents obtained by NLU, which they attached to
February 27, 1940 even in proceedings of this company which supplies the entitled to a new their petition with the SC, were evidence so inaccessible to them at
National Labor character. The first of these Philippine Army. Due to alleged trial. the time of the trial that even with the exercise of due diligence
Union v. rights is the right to a hearing, shortage of leather, Toribio caused they could not be expected to have obtained them and offered as
which includes the right of the the lay off of a number of his evidence in the Court of Industrial Relations. Further, the attached
party interested or affected to employees. However, the National documents and exhibits are of such far-reaching importance and
present his own case and Labor Union, Inc. (NLU) questioned effect that their admission would necessarily mean the modification
submit evidence in support the validity of said lay off as it and reversal of the judgment rendered (said newly obtained records
thereof. Not only must the averred that the said employees include books of business/inventory accounts by Ang Tibay which
party be given an opportunity laid off were members of NLU were not previously accessible but already existing), hence, the NLU
to present his case and to while  no members of the rival should be granted new trial to comply with due process.
adduce evidence tending to labor union (National Worker’s
establish the rights which he Brotherhood) were laid off. NLU The Court of Industrial Relations is not narrowly constrained by
asserts but the tribunal must claims that NWB is a company technical rules of procedure, and Commonwealth Act No. 103
consider the evidence dominated union and Toribio was requires it to act according to justice and equity and substantial
presented. While the duty to merely busting NLU. merits of the case, without regard to technicalities or legal evidence
deliberate does not impose the but may inform its mind in such manner as it may deem just and
obligation to decide right, it The case reached the Court of equitable (Goseco vs. Court of Industrial Relations et al., G. R. No.
does imply a necessity which Industrial Relations (CIR) where 46673). The fact, however, that the Court of Industrial Relations
cannot be disregarded, Toribio and NWB won. Eventually, may be said to be free from the rigidity of certain procedural
namely, that of having NLU went to the Supreme Court requirements does not mean that it can, in justiciable cases coming
something to support its invoking its right for a new trial on before it, entirely ignore or disregard the fundamental and essential
decision. Not only must there the ground of newly discovered requirements of due process in trials and investigations of an
be some evidence to support a evidence. The Supreme Court administrative character.
finding or conclusion, but the agreed with NLU. The Solicitor
evidence must be substantial. General, arguing for the CIR, filed a
The decision must be rendered motion for reconsideration.
on the evidence presented at
the hearing, or at least
contained in the record and
disclosed to the parties
affected. The Court of
Industrial Relations or any of
its judges, therefore, must act
on its or his own independent
consideration of the law and
facts of the controversy, and
not simply accept the views of
a subordinate in arriving at a
decision. The Court of
Industrial Relations should, in
all controversial questions,
render its decision in such a
manner that the parties to the
proceeding can know the
various issues involved, and
the reasons for the decisions
rendered. The performance of
this duty is inseparable from
the authority conferred upon
it.
OCA v. PASCUAL Corruption; Before any member of the One CEFERINO TIGAS wrote a W/N the evidence NO. Respondent was not afforded right to open trial wherein
259 SCRA 604; Administrative Judiciary could be faulted, it letter, addressed to OCA of SC, presented against respondent can confront the witnesses against him and present
JULY 29, 1996 case against should be only after due charging that irregularities and the judge were evidence in his defense. Only bases for the Report and
Judge Filomeno investigation and after corruption were being committed strong enough to Recommendation submitted consist of: The Complaint, the Answer,
Pascual presentation of competent by the RESPONDENT Presiding convict him. the Memorandum of the respondent, and the transcript of
evidence derived from direct Judge of MTC. Letter was referred stenographic notes of the hearing of the bribery case of respondent
knowledge, especially since the to NBI for “discreet investigation” judge at the Sandiganbayan. Before any member of the Judiciary
charge is penal in character. of respondent. In affidavit, Cruz could be faulted, it should be only after due investigation and after
declared that he was the accused presentation of competent evidence derived from direct
in a criminal case for Frustrated knowledge, especially since the charge is penal in character.
Murder. Respondent judge
decided that the crime he
committed was only physical
injuries. Cruz made to understand
that, in view of such action, Cruz
was to give him P2,000.
Respondent judge also believed to
be a drunkard. NBI entrapped
Respondent judge with help of
Cruz, for which reason, the judge
was thought to have been caught
in flagrante delicto. Result of
investigation and Respondent
referred to the Inquest Prosecutor
of the Office of the Special
Prosecutor, Ombudsman, with the
recommendation that he be
charged and prosecuted for
Bribery (Art. 210 RPC)
Executive Judge NATIVIDAD G.
DIZON submitted report and
recommendation that Respondent
judge be penalized for violation of
Canons 2 and 3 of Code of Judicial
Conduct (A Judge should avoid
impropriety and the appearance of
impropriety in all activities and a
judge should perform official
duties honestly, and with
impartiality and diligence).
LUMIQUED v. Waived right to The right to counsel, which Arsenio P. Lumiqued was the W/N the due NO. In the case at bar, petitioners invoke the right of an accused in
EXEVEA counsel by cannot be waived unless the Regional Director of the process clause criminal proceedings to have competent and independent counsel
GR No. 117565. Lumiqued in waiver is in writing and in the Department of Agrarian Reform — encompass the of his own choice. Lumiqued, however, was not accused of any
November 18, alleged presence of counsel, is a right Cordillera Autonomous Region right to be assisted crime in the proceedings below. The investigation conducted by the
1997 corruption afforded a suspect or an (DAR-CAR) until President Fidel V. by counsel during committee created by Department Order No. 145 was for the
accused during custodial Ramos dismissed him from that an administrative purpose of determining if he could be held administratively liable
investigation. It is not an position. inquiry under the law for the complaints filed against him.
absolute right and may, thus, The dismissal was the aftermath of The essence of due process is simply the opportunity to explain
be invoked or rejected in a three complaints filed by DAR-CAR one's side. One may be heard, not solely by verbal presentation but
criminal proceeding and, with Regional Cashier and private also, and perhaps even much more creditably as it is more
more reason, in an respondent Jeannette Obar- practicable than oral arguments, through pleadings. An actual
administrative inquiry. Zamudio with the Board of hearing is not always an indispensable aspect of due process. As
Discipline of the DAR. long as a party was given the opportunity to defend his interests in
The first affidavit-complaint dated due course, he cannot be said to have been denied due process of
November 16, 1989, charged law, for this opportunity to be heard is the very essence of due
Lumiqued with malversation process. Moreover, this constitutional mandate is deemed satisfied
through falsification of official if a person is granted an opportunity to seek reconsideration of the
documents (93 counts). action or ruling complained of. Lumiqued's appeal and his
In her second affidavit-complaint subsequent filing of motions for reconsideration cured whatever
dated November 22, 1989, private irregularity attended the proceedings conducted by the committee.
respondent accused Lumiqued The committee's findings pinning culpability for the charges of
with violation of COA rules and dishonesty and grave misconduct upon Lumiqued were not, as
regulations, alleging that during shown above, fraught with procedural mischief. Its conclusions
the months of April, May, July, were founded on the evidence presented and evaluated as facts.
August, September and October Well-settled in our jurisdiction is the doctrine that findings of fact of
1989, he made unliquidated cash administrative agencies must be respected as long as they are
advances in the total amount of supported by substantial evidence even if such evidence is not
P116,000.00. Lumiqued overwhelming or preponderant. The quantum of proof necessary
purportedly defrauded the for a finding of guilt in administrative cases is only substantial
government. The third affidavit- evidence or such relevant evidence as a reasonable mind might
complaint dated December 15, accept as adequate to support a conclusion.
1989, charged Lumiqued with
oppression and harassment.
Explaining why a vulcanizing shop
issued a gasoline receipt,
Lumiqued said that he and his
companions were cruising along
Santa Fe, Nueva Vizcaya on their
way to Ifugao when their service
vehicle ran out of gas. Since it was
almost midnight, they sought the
help of the owner of a vulcanizing
shop who readily furnished them
with the gasoline they needed. The
vulcanizing shop issued its own
receipt so that they could
reimburse the cost of the gasoline.
Domingo Lucero, the owner of said
vulcanizing shop, corroborated this
explanation in an affidavit dated
June 25, 1990. With respect to the
accusation that he sought
reimbursement in the amount of
P660.00 for one vulcanizing job,
Lumiqued submitted that the
amount was actually only P6.60.
Any error committed in posting
the amount in the books of the
Regional Office was not his
personal error or accountability.
Committee hearings on the
complaints were conducted on
July 3 and 10, 1992, but Lumiqued
was not assisted by counsel. On
the second hearing date, he
moved for its resetting to July 17,
1992, to enable him to employ the
services of counsel. The committee
granted the motion, but neither
Lumiqued nor his counsel
appeared on the date he himself
had chosen, so the committee
deemed the case submitted for
resolution.
FABELLA v. CA Teacher’s Due process of law requires On September 17, 1990, then W/N respondent NO. In administrative proceedings, due process has been recognized
G.R. No. 110379. walkouts and notice and hearing. Hearing, on DECS Secretary Isidro Cariño Court of Appeals to include the following: (1) the right to actual or constructive notice
November 28, strikes the other hand, presupposes a issued a return to work order to all committed grave of the institution of proceedings which may affect a respondent's
1997 competent and impartial public school teachers who had abuse of discretion legal rights; (2) a real opportunity to be heard personally or with the
tribunal. The right to be heard participated in walkouts and in holding in effect assistance of counsel, to present witnesses and evidence in one's
and, ultimately, the right to strikes. Secretary Cariño filed that private favor, and to defend one's rights; (3) a tribunal vested with
due process of law lose administrative charges against the respondents were competent jurisdiction and so constituted as to afford a person
meaning in the absence of an striking teachers due to violation denied due charged administratively a reasonable guarantee of honesty as well
independent, competent and of civil service laws such as grave process of law. as impartially; and (4) a finding by said tribunal which is supported
impartial tribunal. misconduct, gross neglect of duty, by substantial evidence submitted for consideration during the
absence without leave. The hearing or contained in the records or made known to the parties
Secretary also placed the teachers affected.
under preventive suspension. The In the present case, the various committees formed by DECS to hear
DECS Secretary through the the administrative charges against private respondents did not
Solicitor General, contended that include "a representative of the local or, in its absence, any existing
in accordance with the doctrine of provincial or national teacher's organization" as required by Section
primary resort, the trial court 9 of RA 4670. Accordingly, these committees were deemed to have
should not interfere in the no competent jurisdiction. Thus, all proceedings undertaken by
administrative proceedings. them were necessarily void. They could not provide any basis for
Meanwhile, the DECS investigating the suspension or dismissal of private respondents. The inclusion of
committee rendered a decision a representative of a teachers' organization in these committees
finding the striking teachers guilty was indispensable to ensure an impartial tribunal. It was this
as charged and ordered their requirement that would have given substance and meaning to the
dismissal. The trial court also right to be heard. Indeed, in any proceeding, the essence of
dismissed the petition for procedural due process is embodied in the basic requirement of
certiorari and mandamus for lack notice and a real opportunity to be heard.
of merit. The trial court held that Mere membership of said teachers in their respective teachers'
Republic Act No. 4670, otherwise organizations does not ipso facto make them authorized
known as the "Magna Carta for representatives of such organizations as contemplated by Section 9
Public School Teachers," is the of RA 4670. Under this section, the teachers' organization possesses
primary law that governs the the right to indicate its choice of representative to be included by
conduct of investigation in the DECS in the investigating committee. Such right to designate
administrative cases filed against cannot be usurped by the secretary of education or the director of
public school teachers, with Pres. public schools or their underlings. In the instant case, there is no
Decree No. 807 as its dispute that none of the teachers appointed by the DECS as
supplemental law. As a result, the members of its investigating committee was ever designated or
committee tasked to investigate authorized by a teachers' organization as its representative in said
the charges filed against the committee.
teachers was illegally constituted
and all acts done by said body
possess no legal color whatsoever.
From this adverse decision of the
trial court, former DECS Secretary
Cariño filed an appeal with the
Court of Appeals. The Court of
Appeals affirmed the trial court's
decision holding in the main that
private respondents were denied
due process in the administrative
proceedings instituted against
them.
SUMMARY 12 administrative Sometime last April 26, 1994, after
DISMISSAL v. complaints attending the birthday party of
TORCITA before PNP Miss Jessie Vasquez, Alex Edwin
G.R. No. 130442. del Rosario, together with Rosita
April 6, 2000 Bistal, Carmen Braganza and
Cristita Dawa boarded Mazda pick
up with plate nr. HHP-808 and
driven by Reynaldo Consejo,
proceeded towards the direction
of Cadiz City. While nearing
Crossing Cadiz in the vicinity of
Sitio Puting Tubig, the
aforementioned Mazda pick-up
driven by Consejo overtook a red
Cortina Ford driven by Major
Lazaro Torcita; That on board the
motor vehicle driven by Torcita
were three females sitted at the
back; That Major Lazaro Torcita
signaled the passengers of the
Mazda pick-up to stop, however,
the driver of the Mazda pick-up
refused to abide by the signal and
instead accelerated and proceeded
to Hda. Aimee without stopping.
That upon reaching Hda. Aimee
Major Lazaro Torcita, entered the
compound and was approached by
two persons in civilian clothes
which prevented him from further
proceeding; Moments after, the
patrol car of Cadiz PNP arrived and
together with Major Torcita,
approached Jesus H. Puey and Alex
Edwin del Rosario, inquiring as to
the identity of the persons who
accosted him; The complainants
alleged that Major Torcita
approached and entered the
compound of Hda. Aimee, very
drunk, with back-up vehicle full of
armed policemen, confronted
Jesus H. Puey and Alex Edwin del
Rosario as who stopped him at the
gate, shouting in a very, very loud
voice, invectives and remarks; That
such act of Major Lazaro Torcita
constitute Conduct Unbecoming of
an Officer not worth of respect; In
his answer, the respondent, Lazaro
R. Torcita, while admitting that he
entered the premises of the
complainants, the same was done
on a regular, lawful and proper
way for he was in the performance
of his official duties in pursuing the
suspect who committed a crime in
his presence;
The Summary Board find that the
only piece of evidence presented
in connection with the incident
which happened near the gate of
the compound is the affidavit of
C/Insp. Torcita and his testimony
given in the hearing of the case
that when he was walking towards
the compound together with his
aide, PO2 Nehru Java, two armed
civilian guards stopped and
threatened him; He identified
himself however, the same had no
effect, and PO2 Java whispered
that there are armed men around
them and that it is dangerous for
them to continue.
OFFICE OF THE Luncheon In administrative cases, the "In the morning of October 14, Whether or not NO. The notation does not deny respondent of her right to due
OMBUDSMAN v. meeting quantum of proof necessary 1998, Carmencita D. Coronel called respondent was process. In administrative proceedings, the essence of due process
CARMENCITA D. for a finding of guilt is for a meeting the officers of the denied of due lies simply in the opportunity to explain one's side or to seek
CORONEL, G.R. substantial evidence; that is, different Water Districts in Lanao process reconsideration of the action or ruling complained of. What is
No. 164460. June such relevant evidence that a del Norte and Lanao del Sur, as proscribed is the absolute lack of notice or hearing.
27, 2006 reasonable mind might accept well as their advisors from the In this case, respondent was given every opportunity to be heard.
as adequate to support a Local Water Utilities Significantly, her intelligible pleadings before the CA and this Court
conclusion Administration. Since it was indicate that she knew the bases for the ombudsman's Decision. In
nearing lunchtime, the group fact, she very ably pinpointed its alleged errors that she thought
opted to continue their meeting at would merit our review. Not having been left in the dark as to how
Marvilla's Store at Barangay Bunu- it was reached, respondent's insistence on a denial of due process
un, Iligan City. The luncheon has no legal leg to stand on.
meeting, attended by more or less In the instant case, the complainant did not present evidence to
ten (10) persons, was presided support his theory that the photocopy of the original duplicate
over by Advisor Rhodora Gumban reflected the true amount, or that OR No. 0736 had indeed been
of the LWUA. As the host of the falsified. That oversight was fatal to the discharge of his burden of
said meeting, [respondent] paid proof. A reasonable mind will not carelessly jump to the conclusion
for the lunch in the amount of that respondent is the guilty party.
(P1,213.00), as shown in cash The complainant's evidence to prove falsification consisted of an
Invoice No. 0736 dated October unauthenticated photocopy of the original duplicate. He could have
14, 1998. obtained an affidavit from the restaurant proprietor or employee
She then claimed for who had issued the receipt, in order to attest to its due execution
reimbursement of her expenses and authenticity. Absent any proof of due execution and
covered, chargeable against the authenticity, the alleged photocopy of the original duplicate of OR
representation and entertainment No. 0736 does not convince us that it is an accurate reflection of the
account of her office. That very actual bill incurred.
same day, the voucher was
approved and [respondent] got
her reimbursement in the amount
of (P1,213.00).
"On November 17, 1998, Pedro C.
Sausal, Jr. was appointed General
Manager of Linamon Water
District. On February 1999, he filed
with the Office of the
Ombudsman-Mindanao a sworn
letter-complaint against herein
[respondent] for dishonesty. The
complaint alleges that
[respondent] falsified the cash
invoice she submitted for
reimbursement by making it
appear that the luncheon bill was
for (P1,213.00) when in fact, it was
only (P213.00), as reflected in the
photocopy of the original duplicate
of cash invoice No. 0736 dated
October 14, 1998
SEC. OF JUSTICE Extradition Due process is comprised of DOJ received from the Department W/N the YES. The evaluation process, just like the extradition proceedings
v. LANTION request; Mark two components — of Foreign Affairs U.S. a request for respondent Mark proper, belongs to a class by itself. It is sui generis. It is not a
G.R. No. 139465. Jimenez substantive due process which the extradition of private Jimenez is entitled criminal investigation, but it is also erroneous to say that it is purely
January 18, 2000 requires the intrinsic validity of respondent Mark Jimenez to the to the basic rights an exercise of ministerial functions. The Court laid down the test of
the law in interfering with the U.S. for violation of Conspiracy to of due process determining whether an administrative body is exercising judicial
rights of the person to his life, Commit Offense, Attempt to Evade over the functions or merely investigatory functions: Adjudication signifies
liberty, or property, and Tax, Fraud by Wire, Radio, or government’s the exercise of power and authority to adjudicate upon the rights
procedural due process which Television, False Statement, and duties under a and obligations of the parties before it. Hence, if the only purpose
consists of the two basic rights Election Contribution in Name of treaty for investigation is to evaluate evidence submitted before it based
of notice and hearing, as well Another. on the facts and circumstances presented to it, and if the agency is
as the guarantee of being During the evaluation process of not authorized to make a final pronouncement affecting the parties,
heard by an impartial and the extradition, the private then there is an absence of judicial discretion and judgment.
competent tribunal respondent, requested the The basic rights of notice and hearing pervade not only in criminal
petitioner, Secretary of Justice, to and civil proceedings, but in administrative proceedings as well.
The rule of pacta sunt furnish him copies of the Non-observance of these rights will invalidate the proceedings.
servanda, one of the oldest extradition request from the U.S. Individuals are entitled to be notified of any pending case affecting
and most fundamental maxims government, that he be given their interests, and upon notice, they may claim the right to appear
of international law, requires ample time to comment regarding therein and present their side and to refute the position of the
the parties to a treaty to keep the extradition request against opposing parties. In a preliminary investigation which is an
their agreement therein in him after he shall have received administrative investigatory proceeding, Section 3, Rule 112 of the
good faith. copies of the requested papers, Rules of Court guarantees the respondent's basic due process rights,
and to suspend the proceeding in granting him the right to be furnished a copy of the complaint, the
the meantime. a􏰋davits, and other supporting documents, and the right to submit
The petitioner denied the request counter-a􏰋davits and other supporting documents within ten days
in consistent with Art. 7 of the RP from receipt thereof. Moreover, the respondent shall have the right
– US Extradition Treaty which to examine all other evidence submitted by the complainant. The
provides that the Philippine right to information is implemented by the right of access to
Government must represent the information within the control of the government (Bernas, The 1987
interests of the U.S. in any Constitution of the Republic of the Philippines, 1996 ed., p. 337).
proceedings arising from an Such information may be contained in official records, and in
extradition request. documents and papers pertaining to official acts, transactions, or
The private respondent filed with decisions.
the RTC against the petitioner Hon. In the case at bar, the papers requested by private respondent
Ralph Lantion a mandamus, a pertain to official government action from the U.S. Government. No
certiorari, and a prohibition to official action from our country has yet been taken. Moreover, the
enjoin the petitioner, the Secretary papers have some relation to matters of foreign relations with the
of DFA, and NBI from performing U.S. Government. Consequently, if a third party invokes this
any acts directed to the extradition constitutional provision, stating that the extradition papers are
of the respondent, for it will be a matters of public concern since they may result in the extradition of
deprivation of his rights to due a Filipino, we are afraid that the balance must be tilted, at such
process of notice and hearing. particular time, in favor of the interests necessary for the proper
functioning of the government. During the evaluation procedure, no
official governmental action of our own government has as yet been
done; hence the invocation of the right is premature. Later, and in
contrast, records of the extradition hearing would already fall under
matters of public concern, because our government by then shall
have already made an official decision to grant the extradition
request. The extradition of a fellow Filipino would be forthcoming.
GOV’T OF USA v. Mark Jimenez Persons to be extradited are Petitioner US government filed this W/N there was a YES. The Supreme Court allowed a direct invocation of its original
PURUGANAN presumed to be fiight risks. Petition for Certiorari under Rule grave abuse of jurisdiction to issue writs of certiorari to settle once and for all the
GR NO.148571 This prima facie presumption 65 assailing the procedure discretion on the issue of bail in extradition proceedings.
SEPTEMBER 24, finds reinforcement in the adopted by the trial court of first part of the RTC In granting the petition, the Supreme Court held that the present
2002 experience of the executive hearing a potential extraditee, judge to set the extradition case validates the premise that persons sought to be
branch: nothing short of Mark Jimenez, before issuing a hearing for the extradited have a propensity to free. Prior acts of respondent
confinement can ensure that warrant for his arrest under issuance of the eloquently speak of his aversion to the processes in the requesting
the accused will not free the Section 6 of PD No. 1069. warrant of arrest state, as well as his predisposition to avoid them at all costs.
jurisdiction of the requested Petitioner contended that the when it was There is no requirement to notify and to hear the accused before
state in order to thwart their procedure gives Jimenez notice to already evident the issuance of a warrant of arrest under the Constitution which
extradition to the requesting escape and to avoid extradition. from the petition requires only an examination under oath or affirmation of
state. Petitioner also assailed the trial for extradition complainants and the witnesses they may produce; and that since
court's granting of Jimenez's itself and its accused were allowed to be heard and to present evidence at this
prayer for bail, which allows him supporting early stage, the procedure could convert the determination of a
to go on provisional liberty while documents that a prima facie case into a full-blown trial, which is discordant with the
extradition proceedings are prima facie finding rationale for the entire system and anathema to the summary
pending. did exist and he nature of extraditions.
Petitioner no longer filed a Motion may issue a The Court also held that extraditee's immediate detention prior to
for Reconsideration in the warrant for the his being heard does not violate the due process clause; that the
Extradition Court, but resorted immediate arrest right to bail applies only in ordinary criminal proceedings; but that
directly to the Supreme Court of the accused in extradition proceedings, after a potential extraditee has been
instead of the Court of Appeals to arrested, bail may be applied for and granted as an exception.
obtain relief. The present extradition case further validates the premise that
persons sought to be extradited have a propensity to free. Indeed,
extradition hearings would not even begin, if only the accused were
willing to submit to trial in the requesting country. Prior acts of
herein respondent — (1) leaving the requesting state right before
the conclusion of his indictment proceedings there; and (2)
remaining in the requested state despite learning that the
requesting state is seeking his return and that the crimes he is
charged with are bailable — eloquently speak of his aversion to the
processes in the requesting state, as well as his predisposition to
avoid them at all cost: These circumstances point to an ever-
present, underlying high risk of flight. He has demonstrated that he
has the capacity and the will to free. Having fled once, what is there
to stop him, given sufficient opportunity, from fleeing a second
time?
GOV’T OF PH & HKG: An extradition proceeding On January 30, 1995, Philippines W/N trial court YES. The time-honored principle of pacta sunt servanda demands
HONGKONG v. Agreement for being sui generis, the standard and Hong Kong signed an committed grave that the Philippines honor its obligations under the Extradition
OLALIA the surrender of of proof required in granting or "Agreement for the Surrender of abuse of discretion Treaty it entered into with the Hong Kong Special Administrative
GR NO. 153675 accused and denying bail can neither be the Accused and Convicted Persons." It amounting to lack Region. Failure to comply with these obligations is a setback in our
APRIL 19, 2007 convicted proof beyond reasonable took effect on June 20, 1997. or excess of foreign relations and defeats the purpose of extradition. However, it
persons doubt in criminal cases nor the HESAI jurisdiction in does not necessarily mean that in keeping with its treaty
standard of proof of On July 1, 1997, Hong Kong admitting private obligations, the Philippines should diminish a potential extraditee's
preponderance of evidence in reverted back to the People's respondent to bail rights to life, liberty, and due process. More so, where these rights
civil cases. Republic of China and became the are guaranteed, not only by our Constitution, but also by
Hong Kong Special Administrative international conventions, to which the Philippines is a party. We
Region. should not, therefore, deprive an extraditee of his right to apply for
Private respondent Muñoz was bail, provided that a certain standard for the grant is satisfactorily
charged before the Hong Kong met.
Court with 3 counts of the offense While administrative in character, the standard of substantial
in violation of the Prevention of evidence used in administrative cases cannot likewise apply given
Bribery Ordinance of Hong Kong. the object of extradition law which is to prevent the prospective
On September 13, 1999, the DOJ extraditee from fleeing our jurisdiction. In his Separate Opinion in
received from the Hong Kong Purganan, then Associate Justice, now Chief Justice Reynato S.
Department of Justice a request Puno, proposed that a new standard which he termed "clear and
for the provisional arrest of private convincing evidence" should be used in granting bail in extradition
respondent. The DOJ then cases. According to him, this standard should be lower than proof
forwarded the request to the beyond reasonable doubt but higher than preponderance of
National Bureau of Investigation evidence. The potential extraditee must prove by "clear and
(NBI) which, in turn, filed with the convincing evidence" that he is not a flight risk and will abide with
RTC of Manila, Branch 19 an all the orders and processed of the extradition court.
application for the provisional In this case, there is no showing that private respondent presented
arrest of private respondent. evidence to show that he is not a flight risk . Consequently, this case
Petitioner filed with the RTC of should be remanded to the trial court to determine whether private
Manila a petition for the respondent may be granted bail on the basis of "clear and
extradition of private respondent. convincing evidence."
After hearing, or on October 8,
2001, Judge Bernardo, Jr. issued an
Order denying the petition for bail,
holding that there is no Philippine
law granting bail in extradition
cases and that private respondent
is a high "flight risk."
OCAMPO v. Communists A preliminary investigation is Petitioners were charged with the W/N the NO. The essence of due process is reasonable opportunity to be
ABANDO not a casual affair. It is murder of the victims found in a petitioners right to heard and submit evidence in support of one’s defense. Thus, one
GR No. 176830; conducted to protect the mass graveyard with the crime of due process was who has been afforded a chance to present one’s own side of the
February 11, 2014 innocent from the rebellion. They claimed that copies violated story cannot claim denial of due process.
embarrassment, expense, and of the subpoena the complaint and Section 3, Rule 112 of RoC allows prosecutor Vivero to resolve the
anxiety of a public trial. In the other supporting documents never complaint based on the evidence before him if a respondent could
context of a PI, the right to due reached them so that they were not be subpoenaed. As long as efforts to reach a respondent were
process of law entails the denied due process during the made, and he was given an opportunity to present countervailing
opportunity to be heard. It preliminary investigation. evidence, the preliminary investigation remains valid. It was only
serves to accord an Prior receiving the resolution, because a majority of them could no longer be found at their last
opportunity for the Ocampo filed an Ex Parte Motion known addresses that they were not served copies. The rule was
presentation of the to set case for clarificatory meant to foil underhanded attempts of a respondent to delay the
respondent’s side with regard hearing. prosecution of offenses.
to the accusation. Afterwards, Judge Obando found probable
the investigating officer shall cause and ordered the issuance of
decide whether the allegations warrants of arrest against them
and defenses lead to a without bail.
reasonable belief that a crime
has been committed, and that
it was the respondent who
committed it. Otherwise, the
investigating officer is bound to
dismiss the complaint.
ESTRADA v. Jinggoy Estrada; The rights of respondent On 25 November 2013, the W/N Estrada was NO. First. There is no law or rule which requires the Ombudsman to
OMBUDSMAN Furnish copies of Estrada in the conduct of the Ombudsman served upon Sen. denied due furnish a respondent with copies of the counter-affidavits of his co-
GR No. 212140- counter-affidavits preliminary investigation Estrada a copy of the complaint process of law. respondents.
41; January 21, of other depend on the rights granted filed by the NBI and Atty. Baligod Sen. Estrada claims that the denial of his Request for the counter-
2015 respondents to him by law and these cannot for Plunder. And on 3 December affidavits of his co-respondents violates his constitutional right to
be based on whatever rights he 2013, the Ombudsman served due process. Sen. Estrada, however, fails to specify a law or rule
believes that he is entitled to upon Sen. Estrada another which states that it is a compulsory requirement of due process in a
or those that may be derived complaint for the crime of preliminary investigation that the Ombudsman furnish a respondent
from the phrase "due process plunder. with the counter-affidavits of his co-respondents.
of law." Second, it should be underscored that the conduct of a preliminary
18 of Sen. Estrada’s co- investigation is only for the determination of probable cause, and
respondents in the two complaints “probable cause merely implies probability of guilt and should be
filed their counter-affidavits determined in a summary manner. A preliminary investigation is not
between 9 December 2013 and 14 a part of the trial and it is only in a trial where an accused can
March 2014. Sen. Estrada filed his demand the full exercise of his rights, such as the right to confront
request to be furnished with and cross-examine his accusers to establish his innocence.” Thus,
copies of counter affidavits of the the rights of a respondent in a preliminary investigation are limited
other respondents, affidavits of to those granted by procedural law.
new witnesses and other filings.
Sen. Estrada’s request was made A preliminary investigation is defined as an inquiry or proceeding for
"pursuant to the right of a the purpose of determining whether there is sufficient ground to
respondent ‘to examine the engender a well-founded belief that a crime cognizable by the
evidence submitted by the Regional Trial Court has been committed and that the respondent is
complainant which he may not probably guilty thereof, and should be held for trial. The quantum of
have been furnished’ and to ‘have evidence now required in preliminary investigation is such evidence
access to the evidence on record’ sufficient to “engender a well-founded belief” as to the fact of the
based on section 3[b], Rule 112 of commission of a crime and the respondent’s probable guilt thereof.
the Rules of Court. A preliminary investigation is not the occasion for the full and
exhaustive display of the parties’ evidence; it is for the presentation
The Ombudsman issued an of such evidence only as may engender a well-grounded belief that
assailed order denying the motion an offense has been committed and that the accused is probably
of Estrada in response to his guilty thereof. We are in accord with the state prosecutor’s findings
request stating that under the in the case at bar that there exists prima facie evidence of
Rules on Criminal Procedure and petitioner’s involvement in the commission of the crime, it being
Rules of Procedure of the Office of sufficiently supported by the evidence presented and the facts
the Ombudsman, he is not entitled obtaining therein.
to be furnished of the copy of all Probable cause can be established with hearsay evidence, as long as
the filings of the respondents. there is substantial basis for crediting the hearsay. Hearsay evidence
On March 28, 2014, the is admissible in determining probable cause in a preliminary
Ombudsman issued a Joint investigation because such investigation is merely preliminary, and
Resolution which found probable does not finally adjudicate rights and obligations of parties.
cause to indict Estrada and his co- However, in administrative cases, where rights and obligations are
respondents with one count of finally adjudicated, what is required is “substantial evidence” which
plunder and 11 counts of violation cannot rest entirely or even partially on hearsay evidence.
of Section 3(e) of RA No. 3019. Substantial basis is not the same as substantial evidence because
substantial evidence excludes hearsay evidence while substantial
basis can include hearsay evidence.
GUZMAN v. NU
NON v. JUDGE Mass actions Petitioners, students in private W/N the school The Supreme Court ruled that the trial court cannot anchor the
DAMES against school respondent Mabini Colleges, Inc. has the right not “Termination of Contract” theory the contract between the school
were not allowed to re-enroll by to re-admit the and the student is not an ordinary contract. It is imbued with public
the school for the academic year interest, considering the high priority given by the Constitution to
1988-1989 for leading or petitioners. education and the grant to the State of supervisory and regulatory
participating in student mass powers over all educational institutions. The authority for schools to
actions against the school in the refuse enrollment to a student on the ground that his contract,
W/N School’s
preceding semester. The subject of which has a term of one semester, has already expired, cannot be
Academic
the protests is not, however, made justified. Still, institutions' discretion on the admission and
Freedom a
clear in the pleadings. Petitioners enrollment of students as a major component of the academic
Ground for
filed a petition in the court a quo freedom guaranteed to institutions of higher learning. The right of
Denying Students'
seeking their readmission or re- an institution of higher learning to set academic standards,
Rights.
enrollment to the school, but the however, cannot be utilized to discriminate against students who
trial court dismissed the petition exercise their constitutional rights to speech and assembly, for
REFERRING to a ruling in Alcuaz vs. otherwise there will be a violation of their right to equal protection.
PSBA allowing schools to bar the Thus, an institution of learning has a contractual obligation to afford
readmission or re-enrollment of its students a fair opportunity to complete the course they seek to
students on the ground of pursue. However, when a student commits a serious breach of
termination of contract. A motion discipline or fails to maintain the required academic standard, he
for reconsideration was filed, but forfeits his contractual right; and the court should not review the
this was denied by the trial court. discretion of university authorities. Excluding students because of
Hence, petitioners filed the instant failing grades when the cause for the action taken against them
petition for certiorari with prayer undeniably related to possible breaches of discipline not only is a
for preliminary mandatory denial of due process but also constitutes a violation of the basic
injunction tenets of fair play. Further, the failures in one or two subjects by
some cannot be considered marked academic deficiency. Neither
can the academic deficiency be gauged from the academic
standards of the school due to insufficiency of information. Herein,
the students could have been subjected to disciplinary proceedings
in connection with the mass actions, but the penalty that could have
been imposed must be commensurate to the offense committed
and it must be imposed only after the requirements of procedural
due process have been complied with (Paragraph 145, Manual of
Regulations for Private Schools). But this matter of disciplinary
proceedings and the imposition of administrative sanctions have
become moot and academic; as the students have been refused
readmission or re-enrollment and have been effectively excluded
from for 4 semesters, have already been more than sufficiently
penalized for any breach of discipline they might have committed
when they led and participated in the mass actions that resulted in
the disruption of classes. To still subject them to disciplinary
proceedings would serve no useful purpose and would only further
aggravate the strained relations between the students and the
officials of the school which necessarily resulted from the heated
legal battle.

Court's unequivocal statement in Villar that the right of an


institution of higher learning to set academic standards cannot be
utilized to discriminate against students who exercise their
constitutional rights to speech and assembly, for otherwise there
win be a violation of their right to equal protection. It does not
appear that the petitioners were afforded due process, in the
manner expressed in Guzman, before they were refused re-
enrollment. In fact, it would appear from the pleadings that the
decision to refuse them re-enrollment because of failing grades was
a mere afterthought. It is not denied that what incurred the ire of
the school authorities was the student mass actions conducted in
February 1988 and which were led and/or participated in by
petitioners. Certainly, excluding students because of failing grades
when the cause for the action taken against them undeniably
related to possible breaches of discipline not only is a denial of due
process but also constitutes a violation of the basic tenets of fair
play.

ADMU v. Hazing The Minimum standards to be Leonardo H. Villa, a first year law W/N the NO. Corollary to their contention of denials of due process is their
CAPULONG satisfied in the imposition of student of Petitioner University, respondent argument that it is Ang Tibay case and not the Guzman case which
disciplinary sanctions in died of serious physical injuries at students were is applicable in the case at bar. Though both cases essentially deal
academic institutions, such as Chinese General Hospital after the denied due with the requirements of due process, the Guzman case is more
petitioner university herein, initiation rites of Aquila Legis. process apropos to the instant case, since the latter deals specifically with
thus: Bienvenido Marquez was also the minimum standards to be satisfied in the imposition of
hospitalized at the Capitol Medical disciplinary sanctions in academic institutions, such as petitioner
(1) the students must be Center. Petitioner Dean Cynthia university herein, thus:
informed in WRITING of the del Castillo created a Joint
nature and cause of any Administration- Faculty-Student (1) the students must be informed in writing of the nature and
accusation against them; (2) Investigating Committee which cause of any accusation against them; (2) that they shall have the
that they shall have the right to was tasked to investigate and right to answer the charges against them with the assistance of
answer the charges against submit a report within 72 hours on counsel, if desired: (3) they shall be informed of the evidence
them with the assistance of the circumstances surrounding the against them (4) they shall have the right to adduce evidence in
counsel, if desired: (3) they death of Lennie Villa. Said notice their own behalf; and (5) the evidence must be duly considered by
shall be informed of the also required respondent students the investigating committee or official designated by the school
evidence against them (4) they to submit their written statements authorities to hear and decide the case. The requirements are met.
shall have the right to adduce within twenty-four (24) hours from Respondent students were notified and required to submit written
evidence in their own behalf; receipt. Although respondent statements, and such notices and letters were addressed
and (5) the evidence must be students received a copy of the individually to them. Such notices and letters clearly show that
duly considered by the written notice, they failed to file a respondent students were given ample opportunity to adduce
investigating committee or reply. In the meantime, they were evidence in their behalf and to answer the charges leveled against
official designated by the placed on preventive suspension. them. The requisite assistance of counsel was met when, from the
school authorities to hear and The Investigating Committee found very start of the investigations before the Joint Administration
decide the case. a prima facie case against Faculty-Student Committee, the law firm of Gonzales Batiler and
respondent students for violation Bilog and Associates put in its appearance and filed pleadings in
of Rule 3 of the Law School behalf of respondent students.
Catalogue entitled "Discipline."
Respondent students were then
required to file their written
answers to the formal charge.
Petitioner Dean created a
Disciplinary Board to hear the
charges against respondent
students. The Board found
respondent students guilty of
violating Rules on Discipline which
prohibits participation in hazing
activities. However, in view of the
lack of unanimity among the
members of the Board on the
penalty of dismissal, the Board left
the imposition of the penalty to
the University Administration.
Accordingly, Fr. Bernas imposed
the penalty of dismissal on all
respondent students. Respondent
students filed with RTC Makati a
TRO since they are currently
enrolled. This was granted. A day
after the expiration of the
temporary restraining order, Dean
del Castillo created a Special Board
to investigate the charges of
hazing against respondent
students Abas and Mendoza. This
was requested to be stricken out
by the respondents and argued
that the creation of the Special
Board was totally unrelated to the
original petition which alleged lack
of due process.
UP v. LIGOT-TAN
LAO GI v. CA
DOMINGO v.
SCHEER
PHILCOMSAT v.
ALCUAZ
GLOBE TELECOM
v. NTC
G.R. No. 143964.
July 26, 2004

CORONA v.
UHPAP
NPC v.
ZOZOBRADO
SALAW v. NLRC
CASTILLO-CO v.
BARBERS
AMERICAN
INTER-FASHION
v. OP
US v TORIBIO
YNOT v. IAC The minimum requirements of SUPRA W/N EO No. 626-A YES. The thrust of his petition is that the executive order is
due process are notice and is a violation of unconstitutional insofar as it authorizes outright confiscation of the
hearing which, generally Substantive Due carabao or carabeef being transported across provincial boundaries.
speaking, may not be Process. His claim is that the penalty is invalid because it is imposed without
dispensed with because they according the owner a right to be heard before a competent and
are intended as a safeguard impartial court as guaranteed by due process.
against official arbitrariness. The closed mind has no place in the open society. It is part of the
sporting Idea of fair play to hear "the other side" before an opinion
is formed or a decision is made by those who sit in judgment.
Obviously, one side is only one-half of the question; the other half
must also be considered if an impartial verdict is to be reached
based on an informed appreciation of the issues in contention. It is
indispensable that the two sides complement each other, as unto
the bow the arrow, in leading to the correct ruling after examination
of the problem not from one or the other perspective only but in its
totality. A judgment based on less that this full appraisal, on the
pretext that a hearing is unnecessary or useless, is tainted with the
vice of bias or intolerance or ignorance, or worst of all, in repressive
regimes, the insolence of power.
In the instant case, the carabaos were arbitrarily confiscated by the
police station commander, were returned to the petitioner only after
he had filed a complaint for recovery and given a supersedes bond
of P12,000.00, which was ordered confiscated upon his failure to
produce the carabaos when ordered by the trial court. The
executive order defined the prohibition, convicted the petitioner
and immediately imposed punishment, which was carried out
forthright. The measure struck at once and pounced upon the
petitioner without giving him a chance to be heard, thus denying
him the centuries-old guaranty of elementary fair play.
CHURCHILL v.
RAFFERTY
PEOPLE v.
FAJARDO
ERMITA-MANILA
HOTEL v. CITY OF
MANILA
WHITE LIGHT
CORP. v. CITY OF
MANILA
BALACUIT v. CFI
CARLOS
SUPERDRUG v.
DSWD
NDCNA v. PVB
AGUSTIN v. EDU Early warning
device
MAGTAJAS v.
PRYCE
PROPERTIES
DANS v. PEOPLE
CORONA v.
UHPAP
PEOPLE v. DE LA
PIEDRA
ESTRADA v.
SANDIGANBAYAN
GSIS v.
MONTECARLOS
MIRASOL v.
DPWH
PEOPLE v. CAYAT
ICHONG v.
HERNANDEZ
ASSOC. OF SMALL
LANDOWNERS v.
SEC. OF
AGRARIAN
REFORM
VILLEGAS v. HIU
CHIONG TSAI
PAO HO
DUMLAO v.
COMELEC
ORMOC SUGAR
CENTRAL v.
ORMOC CITY
BASCO v.
PAGCOR
BINAY v.
DOMINGO
NPC v. DE
GUZMAN
HIMAGAN v.
PEOPLE
TABLARIN v.
GUTIERREZ
LIM v. PACQUING
PHIL JUDGES
ASSOC v. PRADO
SISON v.
ANCHETA
TELEBAP v.
COMELEC
TIU v. CA
LACSON v. EXEC.
SEC
SORIANO v. CA
LOONG v.
COMELEC
INTERNATIONAL
SCHOOL v.
QUISIMBING
DE GUZMAN v.
COMELEC
DIMAPORO v.
HRET
CENTRAL BANK
EMPLOYEES v.
BANGKO
SENTRAL
SERRANO v.
GALLANT
MARITIME
DIMAYUGA v.
OMB
DISINI v. SEC. OF
JUSTICE
BIRAOGO v. PHIL.
TRUTH
COMMISSION
GARCIA v. VAWC
DRILON
VALMONTE v. Checkpoints in Between the inherent right of On 20 January 1987, the National W/N the NO. Petitioners' concern for their safety and apprehension at being
GEN. DE VILLA Valenzuela the state to protect its Capital Region District Command warrantless search harassed by the military manning the checkpoints are not sufficient
existence and promote public (NCRDC) was activated to maintain and seizure grounds to declare the checkpoints as per se illegal.
welfare and an individual's peace and order, the NCRDC without in the No proof has been presented before the Court to show that, in the
right against a warrantless installed checkpoints in various present case is course of their routine checks, the military indeed committed
search which is however parts of Valenzuela, Metro Manila. illegal. specific violations of petitioners' right against unlawful search and
reasonably conducted, the Petitioners aver that, because of seizure or other rights.
former should prevail. the installation of said Petitioner Valmonte's general allegation to the effect that he had
checkpoints, the residents of been stopped and searched without a search warrant by the
The constitutional right against Valenzuela are worried of being military manning the checkpoints, without more, i.e., without
unreasonable searches and harassed and of their safety being stating the details of the incidents which amount to a violation of
seizures is a personal right placed at the arbitrary, capricious his right against unlawful search and seizure, is not sufficient to
invocable only by those whose and whimsical disposition of the enable the Court to determine whether there was a violation of
rights have been infringed, or military manning the checkpoints, Valmonte's right against unlawful search and seizure. Not all
threatened to be infringed. considering that their cars and searches and seizures are prohibited. Those which are reasonable
What constitutes a reasonable vehicles are being subjected to are not forbidden. A reasonable search is not to be determined by
or unreasonable search and regular searches and check-ups, any fixed formula but is to be resolved according to the facts of each
seizure in any particular case is especially at night or at dawn, case.
purely a judicial question, without the benefit of a search The setting up of the questioned checkpoints in Valenzuela (and
determinable from a warrant and/or court order. probably in other areas) may be considered as a security measure to
consideration of the On 9 July 1988, Benjamin Parpon, enable the NCRDC to pursue its mission of establishing effective
circumstances involved. a supply officer of the Municipality territorial defense and maintaining peace and order for the benefit
of Valenzuela, Bulacan, was of the public. Checkpoints may also be regarded as measures to
gunned down allegedly in cold thwart plots to destabilize the government, in the interest of public
blood by the members of the security. In this connection, the Court may take judicial notice of the
NCRDC manning the checkpoint shift to urban centers and their suburbs of the insurgency
along McArthur Highway at movement, so clearly reflected in the increased killings in cities of
Malinta, Valenzuela, for ignoring police and military men by NPA "sparrow units," not to mention the
and/or refusing to submit himself abundance of unlicensed firearms and the alarming rise in
to the checkpoint and for lawlessness and violence in such urban centers, not all of which are
continuing to speed off inspire of reported in media, most likely brought about by deteriorating
warning shots fired in the air. economic conditions ---- which all sum up to what one can rightly
Petitioner Valmonte also claims consider, at the very least, as abnormal times.
that, on several occasions, he had
gone thru these checkpoints
where he was stopped and his car
subjected to search/check-up
without a court order or search
warrant.
GUAZON v. DE Area target The validity of search warrant The 41 petitioners alleged that the W/N the are target YES. The conduct of areal target zoning or saturation drive is a valid
VILLA Zoning was not questioned "saturation drive" or "areal target zoning and the exercise of the military powers of the President. 
zoning" that were conducted in saturation drive is The areal target zonings in this petition were intended to flush out
The Supreme Court upheld. as their place (Tondo Manila) were legal subversives and criminal elements particularly because of the
a valid exercise of the military unconstitutional.  blatant assassinations of public officers and police officials by
powers of the President, the The alleged acts committed during elements supposedly coddled by the communities where the
conduct of “areal target the raid are the following: "drives" were conducted.
zoning” or “saturation drive/s”. Petitioners alleged that there is no Moreover, there is nothing in the Constitution which denies the
[NOTE: In this case, the validity specific target house to search and authority of the Chief Executive, to order police actions to stop
of the search was not directly that there is no search warrant or unabated criminality, rising lawlessness, and alarming communist
questioned; raised in issue warrant of arrest served. Most of activities. 
were the alleged abuses the policemen are in their civilian Where there is large scale mutiny or actual rebellion, the police or
committed by the military clothes and without nameplates or military may go in force to the combat areas, enter affected
personnel who conducted the identification cards. The residents residences or buildings, round up suspected rebels and otherwise
“saturation drives”. In the were rudely roused from their quell the mutiny or rebellion without having to secure search
absence of complainants and sleep by banging on the walls and warrants and without violating the Bill of Rights. 
complaints against specific windows of their houses. They The Constitution grants the Government the power to seek and
actors, no prohibition could be were ordered to strip down to cripple subversive movements which would bring down constituted
issued. However, the their briefs for the police to authority and substitute a regime where individual liberties are
temporarily restrained alleged examine their tattoo marks. The suppressed as a matter of policy in the name of security of the
banging of kicking of doors, residents complained that they're State. 
herding of half-naked men for homes were ransacked, tossing The facts that on twelve occasions between March and November,
examination of tattoo marks, their belongings and destroying 1987 the military conducted the saturation drives in question is a
the violation of residences, their valuables. Some of their fact open to no question. The Solicitor General admits that they, the
even if these are humble money and valuables had saturation drives, had been done, except that they had been done
shanties of squatters, and the disappeared after the "with due regard to human rights.
other alleged acts which are operation. Those who were The question, then, is purely one of law: Are the saturation drives in
shocking to the conscience. detained also suffered mental and question lawful and legitimate? It is also a question that is nothing
The Supreme Court remanded physical torture to extract novel: No, because the arrests were not accompanied by a judicial
the case to the trial court for confessions and tactical warrant. Therefore, the fact that they had been carefully planned,
reception of evidence on the information. executed in coordination with Tondo's barangay officials, and
alleged abuses. undertaken with due courtesy and politeness (which I doubt),will
Respondents said that such not validate them. The lack of a warrant makes them, per se, illegal.
accusations mention I find allusions to the last aborted coup d'etat inapt. In that case, our
above were total lies. The aerial men in uniform had all the right to act amidst crimes being
target zoning was intended to committed in flagrante. The instant case is quite different. There are
flush out subversives and criminal no offenses being committed, but rather, police officers fishing for
elements coddled by the evidence of offenses that may have been committed. As I said, in
communities were the said drives that event, a court warrant is indispensable.
were conducted. They As a general rule, a peace officer cannot act unless he is possessed
averred that they have intelligently of the proper arrest or search warrant. The exception is when a
and carefully planned months criminal offense is unfolding before him, in which case, action is
ahead for the actual operation and justified and necessary. The majority would have the exception to
that local and foreign media joined be simply, the general rule.
the operation to witness and
record such event.
PEOPLE v. ANDRE 4 gift packages to The constitutional proscription Andre Marti went to the booth of W/N an act of a NO. In the absence of governmental interference, the liberties
MARTI Switzerland against unlawful searches and the Manila Packing and Export private individual, guaranteed by the Constitution cannot be invoked against the State.
seizures applies as a restraint Forwarders in the Pistang Pilipino allegedly in This constitutional right refers to the immunity of one's person,
directed only against the Complex, Ermita, Manila, carrying violation of whether citizen or alien, from interference by government. The
government and its agencies with them 4 gift- wrapped appellant's contraband in the case at bar came into possession of the
tasked with the enforcement of packages. constitutional government without the latter transgressing appellant's rights
the law. Thus, it could only be The packages were not inspected rights, be invoked against unreasonable searches and seizures. If the search is made at
invoked against the State to by Anita Reyes, the proprietress, against the state the behest or initiation of the proprietor of a private establishment
whom the restraint against as Marti refused, who assured her for its own and private purposes, as in the case at bar, and without
arbitrary and unreasonable that the packages simply the intervention of police authorities, the right against
exercise of power is imposed contained books, cigars, and unreasonable searches and seizures cannot be invoked for only the
gloves and were gifts to his friend act of private individuals, not law enforcers, is involved. In sum, the
in Zurich. protection against unreasonable searches and seizures cannot be
However, before delivery of extended to acts committed by private individuals so as to bring it
appellant's box to the Bureau of within the ambit of alleged unlawful intrusion by the government.
Customs and/ or Bureau of Posts,
Mr. Job Reyes, proprietor and
husband of Anita Reyes, following
standard operating procedure,
opened the boxes for final
inspection. When he opened
appellant's box, a peculiar odor
emitted therefrom. His curiosity
aroused. He squeezed one of the
bundles allegedly containing
gloves and felt dried leaves inside.
Opening one of the bundles, he
pulled out a cellophane wrapper
protruding from the opening of
one of the gloves. He made an
opening on one of the cellophane
wrappers and took several grams
of the contents thereof. Job Reyes
reported the incident to the NBI
and requested a laboratory
examination of the samples he
extracted from the cellophane
wrapper. It turned out that the
dried leaves were marijuana
flowering tops as certified by the
forensic chemist of the Narcotics
Section of the NBI.

BACHE and CO. v. Corporation A corporation is entitled to On February 24, 1970, Misael P. W/N the search NO.
RUIZ immunity against Vera, Commissioner of Internal warrant issued by Three grounds: 1. Judge Ruiz failed to personally examine the
unreasonable searches and Revenue, wrote a letter addressed Judge Ruiz is valid complainant and his witness.
seizures. A corporation is, after to Judge Vivencio M. Ruiz As applied in the instant case, the reading of the stenographic notes
all, but an association of requesting the issuance of a search W/N Bache and to Judge Ruiz did not constitute sufficient compliance with the
individuals under an assumed warrant against Bache & Co. Co. is entitled to constitutional mandate and the rule; for by that manner Judge Ruiz
name and with a distinct legal (Phils.) for violation of Section protection against did not have the opportunity to observe the demeanor of the
entity. In organizing itself as a 46(a) of the NIRC, in relation to all unreasonable complainant and his witness, and to propound initial and follow-up
collective body it waives no other pertinent provisions thereof, searches and questions which the judicial mind, on account of its training, was in
constitutional immunities particularly Sections 53, 72, 73, seizures the best position to conceive. These were important in arriving at a
appropriate to such body. Its 208 and 209. The said letter sound inference on the all-important question of whether or not
property cannot be taken likewise authorized Revenue there was probable cause
without compensation. It can Examiner Rodolfo de Leon to make 2. The search warrant was issued for more than one specific
only be proceeded against by and file the application for search offense.
due process of law, and is warrant. In the afternoon of the In the instant case, Search Warrant No. 2-M-70 was issued for
protected against unlawful following day, De Leon and his "violation of Sec. 46(a) of the National Internal Revenue Code in
discrimination witness, Arturo Logronio, went to relation to all other pertinent provisions thereof particularly Secs.
the CFI of Rizal. They brought with 53, 72, 73, 208 and 209." As can be seen, the search warrant in
them the following papers: Vera’s question was issued for at least four distinct offenses under the Tax
aforesaid letter-request; an Code.
application for search warrant 3. The search warrant does not particularly describe the things to be
already filled up but still unsigned seized.
by De Leon; an affidavit of A search warrant may be said to particularly describe the things to
Logronio subscribed before De be seized when the description therein is as specific as the
Leon; a deposition in printed form circumstances will ordinarily allow; or when the description
of Logronio already accomplished expresses a conclusion of fact — not of law — by which the warrant
and signed by him but not yet officer may be guided in making the search and seizure; or when the
subscribed; and a search warrant things described are limited to those which bear direct relation to
already accomplished but still the offense for which the warrant is being issued.
unsigned by Judge Ruiz.
At that time, Judge Ruiz was still YES. The tax assessments made by the BIR against Bache & Co. were
hearing a different case. So by entirely, or at least partly, based on the documents seized by virtue
means of a note, he instructed his of Search Warrant No. 2-M-70. Furthermore, the fact that the
Deputy Clerk of Court to take the assessments were made some one and one-half months after the
depositions of De Leon and search and seizure on February 25, 1970, is a strong indication that
Logronio. the documents thus seized served as basis for the assessments.
After the session had adjourned,
Judge Ruiz was informed that the
depositions had already been
taken. The stenographer, upon
request of Judge Ruiz, read to him
her stenographic notes; and
thereafter, Judge Ruiz asked
Logronio to take the oath and
warned him that if his deposition
was found to be false and without
legal basis, he could be charged for
perjury. Judge Ruiz then signed De
Leon’s application for search
warrant and Logronio’s deposition.
Search Warrant No. 2-M-70 was
then sign by Judge Ruiz and
accordingly issued.
February 28, 1970, which was a
Saturday, the BIR agents served
the search warrant against Bache
& Co. at their Ayala Avenue office
in Makati. The company's lawyers
protested the search on the
ground that no formal complaint
or transcript of testimony was
attached to the warrant. The
agents nevertheless proceeded
with their search which yielded six
boxes of documents
STONEHILL v. 42 Search Two points must be stressed in Upon application of the officers of W/N search NO. None of these requirements has been complied with. Indeed,
DIOKNO Warrant connection with Art. III, Section the government (respondent warrants in the same were issued upon applications stating that the natural and
2 of the Constitution: prosecutors), several judges question were juridical persons therein named had committed a "violation of
(a) that no warrant shall (respondent judges) issued a total validly issued and Central Bank Laws, Tariff and Customs Laws, Internal Revenue
issue but upon of 42 search warrants against the articles seized (Code) and Revised Penal Code." No specific offense had been
probable cause to be petitioners & or the corporations were admissible as alleged in said applications. The averments thereof with respect to
determined by the of which they were officers, evidence the offense committed were abstract. As a consequence, it was
judge in the manner directed to any peace officer, to impossible for the judges who issued the warrants to have found the
set forth therein; and search the persons named and/ or existence of a probable cause, for the same presupposes the
(b) that the warrant shall the premises of their offices, introduction of competent proof that the party against whom it is
particularly describe warehouses, and/ or residences, sought has performed particular acts, or committed specific
the things to be and to seize several personal omissions, violating a given provision of our criminal laws.
seized. properties as the "subject of the To uphold the validity of the warrants in question would be to wipe
offense; stolen or embezelled or out completely one of the most fundamental rights guaranteed in
the fruits of the offense," or "used our Constitution, for it would place the sanctity of the domicile and
THE RIGHT AGAINST or intended to be used as the the privacy of communication and correspondence at the mercy of
UNREASONABLE SEARCHES means of committing the offense" the whims, caprice or passion of peace officers. This is precisely the
AND SEIZURES IS PERSONAL. as violation of Central Bank Laws, evil sought to be remedied by the constitutional provision above
Thus, the documents, papers, Tariff and Customs Laws (TCC), quoted — to outlaw the so-called general warrants. It is not
and things seized under the NIRC and the RPC." difficult to imagine what would happen, in times of keen political
alleged authority of the Petitioners alleged that the strife, when the party in power feels that the minority is likely to
warrants in question may be aforementioned search warrants wrest it, even though by legal means.
split into (2) major groups, are null and void, as contravening Thus, the warrants authorized the search for and seizure of records
namely: the Constitution and the Rules of pertaining to ALL business transactions of petitioners herein,
Court — because, inter alia: (1) regardless of whether the transactions were legal or illegal. The
they do not describe with warrants sanctioned the seizure of all records of the petitioners and
(a) those found and seized in
particularity the documents, books the aforementioned corporations, whatever their nature, thus
the offices of the
and things to be seized; (2) cash openly contravening the explicit command of our Bill of Rights —
aforementioned corporations
money, not mentioned in the that the things to be seized be particularly described — as well as
and
warrants, were actually seized; (3) tending to defeat its major objective: the elimination of general
(b) those found seized in the
the warrants were issued to warrants.
residences of petitioners
evidence against the General search warrants are outlawed because they place the
herein.
aforementioned petitioners in sanctity of the domicile and the privacy of communication and
deportation cases <led against correspondence at the mercy of the whims, caprice or passion of
them; (4) the searches and peace officers. The warrants sanctioned the seizure of all records of
seizures were made in an illegal the petitioners and the aforementioned corporations, whatever
manner; and (5) the documents, their nature, thus openly contravening the explicit command of our
papers and cash money seized Bill of Rights-- THAT THE THINGS TO BE SEIZED BE PARTICULARLY
were not delivered to the courts DESCRIBED-- as well as tending to defeat its major objective: the
that issued the warrants, to be elimination of general warrants.
disposed of in accordance with
law. The said petitioners Stonehill,
et.al. filed w/ the SC this original
action for certiorari, prohibition,
mandamus and injunction.
ALVAREZ v. CFI Oath The oath required must refer On June 3, 1936, the chief of the W/N the warrant YES. Section 1, paragraph 3, of Article III of the Constitution, relative
to the truth of the facts within secret service of the Anti-Usury of arrest herein to the bill of rights, provides that "The right of the people to be
the personal knowledge of the Board, of the Department of illegally issued secure in their persons, houses, papers, and effects against
petitioner or his witnesses, Justice, presented to Judge unreasonable searches and seizures shall not be violated, and no
because the purpose thereof is Eduardo Gutierrez David then warrants shall issue but upon probable cause, to be determined by
to convince the committing presiding over the Court of First the judge after examination under oath or affirmation of the
magistrate, not the individual Instance of Tayabas, an affidavit complainant and the witnesses he may produce, and particularly
making the affidavit and alleging that according to reliable describing the place to be searched, and the persons or things to be
seeking the issuance of the information, the petitioner kept in seized." Section 97 of General Orders, No. 58 provides that "A
warrant, of the existence of his house in Infanta, Tayabas, search warrant shall not issue except for probable cause and upon
probable cause .The true test books, documents, receipts, lists, application supported by oath particularly describing the place to
of sufficiency of an affidavit to chits and other papers used by him be searched and the person or thing to be seized." It will be noted
warrant issuance of a search in connection with his activities as that both provisions require that there be not only probable cause
warrant is whether it has been a money- lender, charging usurious before the issuance of a search warrant but that the search
drawn in such a manner that rates of interest in violation of the warrant must be based upon an application supported by oath of
PERJURY could be charged law. In his oath at the end of the the applicant and the witnesses he may produce . In its broadest
thereon and affiant be held affidavit, the chief of the secret sense, an “ OATH” includes any form of attestation by which a
liable for damages caused. It service stated that his answers to party signifies that he is bound in conscience to perform an act
will likewise be noted that the questions were correct to the faithfully and truthfully; and it is sometimes defined as an
section 1, paragraph 3, of best of his knowledge and belief. outward pledge given by the person taking it that his attestation
Article III of the Constitution He did not swear to the truth of or promise is made under an immediate sense of his responsibility
prohibits unreasonable his statements upon his own to God.
searches and seizures. knowledge of the facts but upon
the information received by him Taking into consideration the nature of the articles so described, it
from a reliable person. is clear that no other more adequate and detailed description
could have been given, particularly because it is difficult to give a
particular description of the contents thereof. The description so
made substantially complies with the legal provisions because the
officer of the law who executed the warrant was thereby placed in
a position enabling him to identify the articles, which he did. The
last ground alleged by the petitioner, in support of his claim that
the search warrant was obtained illegally, is that the articles were
seized in order that the Anti-Usury Board might provide itself with
evidence to be used by it in the criminal case or cases which might
be filed against him for violation of the Anti-Usury Law. (fishing
expedition

BURGOS v. CHIEF Judge Cruz-Pano Description of articles sought The "Metropolitan Mail" and "We W/N the search NO. Insufficient as basis for the determination of probable cause is
OF STAFF issued search to be seized cannot be general. Forum” newspapers were warrants could be the statement contained in the joint affidavit of Alejandro M.
warrants for the searched and its office and printing deemed invalid Gutierrez and Pedro U. Tango, "that the evidence gathered and
offices of machines, equipment, when it only collated by our unit clearly shows that the premises above-
“Metropolitan paraphernalia, motor vehicles and specified one mentioned were used and are continuously being used for
Mail” and “We other articles used in the printing, address but subversive activities in conspiracy with, and to promote the
Forum,” during publication and distribution of the searched two objective of, illegal organization such as the Light-a-Fire
which printing said newspapers, as well as places Movement." In mandating that "no warrant shall issue except upon
equipment, numerous papers, documents, probable cause to be determined by the judge,. . . after examination
motor vehicles, books and other written literature under oath or affirmation of the complainant and the witnesses he
documents, alleged to be in the possession and may produce; (Sec. 3, Art. IV, 1973 Constitution) the Constitution
books, etc control of petitioner Jose Burgos, requires no less than personal knowledge by the complainant or his
possessed by Jr. publisher-editor of the "We witnesses of the facts upon which the issuance of a search warrant
Burgos Jr., the Forum" newspaper, were seized may be justified.
publisher-editor, based on the strength of the two The premises searched were the business and printing offices of the
because these [2] search warrants issued by "Metropolitan Mail" and the "We Forum" newspapers. As a
were alleged to respondent Judge Ernani Cruz- consequence of the search and seizure, these premises were
be used in Pano. Petitioners averred that the padlocked and sealed, with the further result that the printing and
subversive search warrant should be declared publication of said newspapers were discontinued. Such closure is in
activities. illegal because: The judge failed to the nature of previous restraint or censorship abhorrent to the
conduct an examination under freedom of the press guaranteed under the fundamental law, (Sec.
oath or affirmation of the 9, Art. IV of the Constitution) and constitutes a virtual denial of
applicant and his witnesses, as petitioners' freedom to express themselves in print. This state of
mandated by the above-quoted being is patently anathematic to a democratic framework where a
constitutional provision as wen as free, alert and even militant press is essential for the political
Sec. 4, Rule 126 of the Rules of enlightenment and growth of the citizenry.||| 
Court. There are two (2) search
warrants issued but pinpointed
only one place where petitioner
Jose Burgos, Jr. was allegedly
keeping and concealing the articles
listed. That the articles belonging
to his co-petitioners Jose Burgos,
Sr., Bayani Soriano and the J.
Burgos Media Services, Inc. were
seized although the warrants were
directed against Jose Burgos, Jr.
Alone. The search warrant was
based only on the affidavits of Col.
Abadilla’s that they conducted
surveillance of the premises could
not have provided sufficient basis
for the finding of a probable cause.

SOLIVEN v. Exclusive and personal Petitioner Luis Beltran contends, W/N the NO. What the Constitution underscores is the exclusive and
MAKASIAR responsibility of judge to among others, that his petitioner’s personal responsibility of the issuing judge to satisfy himself the
discern probable cause. constitutional rights were violated contention is existence of probable cause . In satisfying himself of the existence of
when respondent judge issued a correct probable cause for the issuance of a warrant of arrest, THE JUDGE IS
warrant of arrest against him NOT REQUIRED TO PERSONALLY EXAMINE THE COMPLAINANT AND
FOR WARRANTS OF ARRESTS, without personally examining the HIS WITNESSES.
THE JUDGES ARE NOT complainant and the witnesses, if
REQUIRED TO PERSONALLY any, to determine probable cause. Following established doctrine and procedure, he shall:
EXAMINE THE COMPLAINANT Petitioner contends that the (1)personally evaluate the report and the supporting documents
AND HIS WITNESSES. The Constitution now requires the submitted by the fiscal regarding the existence of probable cause
addition of the word judge to personally examine the and, on the basis thereof, issue a warrant of arrest; or
"personally" after the word complainant and his witnesses in
"determined" and the deletion his determination of probable
(2) if on the basis thereof he finds no probable cause, he may
of the grant of authority by the cause for the issuance of warrants
disregard the fiscal's report and require the submission of
1973 Constitution to issue of arrests.
supporting affidavits of witnesses to aid him in arriving at a
warrants to "other responsible conclusion as to the existence of probable cause.
officers as may be authorized The basis for his contention was
by law", has apparently the fact that the word “personally”
Sound policy dictates this procedure, otherwise judges would be
convinced petitioner Beltran was added after the word
unduly laden with the preliminary examination and investigation of
that the Constitution now “determined”, and the phrase
criminal complaints instead of concentrating on hearing and
requires the judge to “other responsible officers as may
personally examine the be authorized by law” was
complainant and his witnesses omitted. deciding cases filed before their courts.
in the determination of
probable cause for the
issuance of warrants of arrest.
This is not an accurate
interpretation. What the
Constitution underscores is the
exclusive and personal
responsibility of the issuing
judge to satisfy himself the
existence of probable cause. In
satisfying himself of the
existence of probable cause for
the issuance of a warrant of
arrest, the judge is not
required to personally examine
the complainant and his
witnesses.
SILVA v. Thus, Sections 3 and 4, Rule On June 12, 1986, Villamor, Jr., W/N petitioners’ YES. The depositions of the witnesses did not only contain leading
PRESIDING JUDGE 126 of the Rules of Court chief of the PC Narcom right to personal questions but it was also very broad. The questions propounded to
of RTC NEGROS provide for the requisites for Detachment in Dumaguete City, liberty and the witnesses were in fact, not probing but were merely routinary.
the issuance of a search Province of Negros Oriental, filed security of homes The deposition was already mimeographed and all that the
warrant, to wit: an Application for Search Warrant against witnesses had to do was fill in their answers on the blanks provided.
with the RTC against petitioners unreasonable
"SECTION 3. Requisite for Silva. Respondent Judge, on the searches and “The 'probable cause' required to justify the issuance of a search
issuing search warrant. — A same day issued Search seizures as warrant comprehends such facts and circumstances as will induce a
search warrant shall not issue contemplated in cautious man to rely upon them and act in pursuant thereof. Of the
but upon probable cause in Warrant No. 1, directing the police Art. III, Sec. 2 of 8 questions asked, the 1st, 2nd and 4th pertain to identity. The 3rd
the 1987
connection with one specific officers to search the room of
Constitution was and 5th are leading not searching questions. The 6th, 7th and 8th
offense to be determined Marlon Silva in the residence of refer to the description of the personalities to be seized, which is
personally by the judge after Nicomedes Silva for violation of RA violated.
identical to that in the Search Warrant and suffers from the same
examination under oath or 6425 otherwise known as lack of particularity. The examination conducted WAS GENERAL IN
affirmation of the complainant Dangerous Drugs Act of 1972. NATURE AND MERELY REPETITIOUS of the deposition of said
and the witnesses he may During the search conducted by witness. Mere generalization will not suffice and does not satisfy the
produce, and particularly the police officers, they also seized requirements or probable cause upon which a warrant may issue."
describing the place to be money belonging to petitioner
searched and the things to be Antonieta Silva amounting to
seized. P1,231.40. Moreover, a perusal of the deposition of P/Lt. Florencio Angeles
shows that it was too brief and short. RESPONDENT JUDGE DID NOT
EXAMINE HIM 'IN THE FORM OF SEARCHING QUESTIONS AND
"SECTION 4. Examination of Thereafter, Antonieta Silva filed a ANSWERS'. ON THE CONTRARY, THE QUESTIONS ASKED WERE
complainant; record. — The motion for the return of the said LEADING AS THEY CALLED FOR A SIMPLE 'YES' OR 'NO' ANSWER. As
judge must, before issuing the amount on the ground that the held in Quintero vs. NBI, 'the questions propounded by respondent
warrant, personally examine in search warrant only authorized the Executive Judge to the applicant's witness are not sufficiently
the form of searching police officers to seize marijuana searching to establish probable cause. Asking of leading questions
questions and answers, in dried leaves, cigarettes and joint, to the deponent in an application for search warrant, and
conducting of examination in a general manner, would not satisfy
writing and under oath the and that said officers failed or the requirements for issuance of a valid search warrant." Thus, in
complainant and any witnesses refused to make a return of the issuing a search warrant, the judge must strictly comply with the
he may produce on facts said search warrant in violation of constitutional and statutory requirement that he must determine
personally known to them and Sec. 11, Rule 126 of the Rules of the existence of probable cause by personally examining the
attach to the record their Court. applicant and his witnesses in the form of searching questions and
sworn statements together answers. His failure to comply with this requirement constitutes
with any affidavits submitted." grave abuse of discretion.
Moreover, a perusal of the deposition of P/Lt. Florencio Angeles
shows that it was too brief and short. RESPONDENT JUDGE DID NOT
EXAMINE HIM 'IN THE FORM OF SEARCHING QUESTIONS AND
ANSWERS'. ON THE CONTRARY, THE QUESTIONS ASKED WERE
LEADING AS THEY CALLED FOR A SIMPLE 'YES' OR 'NO' ANSWER. As
held in Quintero vs. NBI, 'the questions propounded by respondent
Executive Judge to the applicant's witness are not sufficiently
searching to establish probable cause. Asking of leading questions
to the deponent in an application for search warrant, and
conducting of examination in a general manner, would not satisfy
the requirements for issuance of a valid search warrant." Thus, in
issuing a search warrant, the judge must strictly comply with the
constitutional and statutory requirement that he must determine
the existence of probable cause by personally examining the
applicant and his witnesses in the form of searching questions and
answers. His failure to comply with this requirement constitutes
grave abuse of discretion
LIM v. FELIX Murder; Masbate What constitutes “personal On March 17, 1989, at about 7:30 W/N a Judge NO. In this case, the records of the preliminary investigation
Domestic Airport examination” depends on the o'clock in the morning, at the without conducted in Masbate were still in Masbate when the warrants of
circumstances of each case. vicinity of the airport road of the ascertaining the arrest were issued against the petitioners. There was no basis for
The judge has discretion to be Masbate Domestic Airport, located facts through his the respondent Judge to make his own personal determination
as brief or as detailed in his or at the municipality of Masbate own personal regarding the existence of a probable cause for the issuance of a
her examination. However, to province of Masbate, determination and warrant of arrest as mandated by the Constitution. He could not
be sure, the judge must at least Congressman Moises Espinosa, Sr. relying solely on possibly have known what transpired in Masbate as he had nothing
go beyond the Prosecutor’s and his security escorts, namely the certification or but a certification. Significantly, the respondent Judge denied the
bare certification. Provincial Guards Antonio Cortes, recommendation petitioners' motion for the transmittal of the records on the ground
Gaspar Amaro, and Artemio of a prosecutor that the mere certification and recommendation of the respondent
Fuentes were attacked and killed that a probable Fiscal that a probable cause exists is sufficient for him to issue a
by a lone assassin. An investigation cause exists issue a warrant of arrest. The evidence the petitioners presented to the
of the incident then followed. warrant of arrest judge were documents of recantation (retraction) of witnesses
whose testimonies were used to establish prima facie evidence
On August 29, 1989, the entire against them. Although, the general rule is that recantations are not
records of the case consisting of given much weight in the determination of a case and in the
261 pages were transmitted to the granting of a new trial, the respondent Judge before issuing his own
Provincial Prosecutor of Masbate. warrants of arrest should, at the very least, have gone over the
On September 22, 1989, Fiscal records of the preliminary examination conducted earlier in the
Alfane issued a Resolution which light of the evidence now presented by the concerned witnesses in
affirmed the finding of a prima view of the "political undertones" prevailing in the cases.
facie case against the petitioners
but differed in the designation of
the crime of murder.

Petitioners Vicente Lim, Sr. and


Susana Lim filed with us a verified
petition for change of venue and
was granted to avoid a miscarriage
of justice. (from Masbate to
Makati RTC). The cases were
raffled to Branch 56 presided by
respondent Judge Nemesio S.
Felix.
Petitioners questioned the validity
of the warrant of arrest because it
was not personally determined by
the judge as he relied solely on the
certification or recommendation of
a prosecutor that a probable cause
exists. They reiterated that the
court should conduct a hearing to
determine if there really was
prima facie evidence against them
based on the documents from the
preliminary investigation. This was
denied for lack of merit by
respondent Judge Felix. In this
denial, he stated that the
investigation in Masbate had
already established prima facie
evidence against them, and that
this was confirmed by the
Provincial Prosecutor in Masbate.
Given that they were both
competent officers, he found no
reason to re-examine the
documents. Petitioners then filed
this case questioning respondent
judge’s order.
MATA v. BAYONA Jai Alai Game Mere affidavits of the Soriano Mata was accused under W/N search NO. . The judge's insistence that she examined the complainants
complainant and his witnesses Presidential Decree (PD) 810, as warrant was valid under oath has become dubious by petitioner's claim that at the
are thus not sufficient. The amended by PD 1306, the particular time when he examined all the relevant papers
examining Judge has to take information against him alleging connected with the issuance of the questioned search warrant,
depositions in writing of the that Soriano Mata offered, took after he demanded the same from the lower court since they were
complainant and the witnesses and arranged bets on the Jai Alai not attached to the records, he did not find any certification at the
he may produce and to attach game by “selling illegal tickets back of the joint affidavit of the complainants .
them to the record . Such known as ‘Masiao tickets’ without
written deposition is necessary any authority from the Philippine As stated earlier, before he filed his motion to quash the search
in order that the Judge may be Jai Alai & Amusement Corporation
able to properly determine the or from the government warrant and for the return of the articles seized, he was furnished,
existence or non-existence of authorities concerned.” upon his request, certified true copies of the said affidavits by the
the probable cause, to hold Clerk of Court but which certified true copies do not bear any
liable for perjury the person Petitioner claims that during the certification at the back. Petitioner likewise claims that his xerox
giving it if it will be found later hearing of the case, he discovered copy of the said joint affidavit obtained at the outset of this case
that his declarations are false. that nowhere from the records of does not show also the certification of respondent judge. This doubt
the said case could be found the becomes more confirmed by respondent Judge's own admission,
The examination or search warrant and other while insisting that she did examine thoroughly the applicants, that
investigation which must be pertinent papers connected to the "she did not take the deposition of Mayote and Goles because to
under oath may not be in issuance of the same, so that he have done so would be to hold a judicial proceeding which will be
public. It may even be held in had to inquire from the City Fiscal open and public", such that, according to her, the persons subject of
the secrecy of his chambers. its whereabouts, and to which the intended raid will just disappear and move his illegal operations
Far more important is that the inquiry Judge Josephine K. Bayona somewhere else. Could it be that the certification was made
examination or investigation is replied, “it is with the court”. The belatedly to cure the defect of the warrant? Be that as it may, there
not merely routinary but one Judge then handed the records to was no "deposition in writing" attached to the records of the case in
that is thorough and elicit the the Fiscal who attached them to palpable disregard of the statutory prohibition heretofore quoted.
required information. To the records. This led Mata to file a Respondent Judge impresses this Court that the urgency to stop the
repeat, it must be under oath motion to quash and annul the illegal gambling that lures every man, woman and child, and even
and must be in writing. search warrant and for the return the lowliest laborer who could hardly make both ends meet justifies
of the articles seized, citing and her action. She claims that in order to abate the proliferation of this
invoking, among others, Section 4 illegal "masiao" lottery, she thought it more prudent not to conduct
of Rule 126 of the Revised Rules of the taking of deposition which is done usually and publicly in the
Court. The motion was denied by court room.
the Judge, stating that the court
has made a thorough investigation
and examination under oath of
Bernardo U. Goles and Reynaldo T.
Mayote, members of the
Intelligence Section of 352nd PC
Co./Police District II INP; that in
fact the court made a certification
to that effect; and that the fact
that documents relating to the
search warrant were not attached
immediately to the record of the
criminal case is of no moment,
considering that the rule does not
specify when these documents are
to be attached to the records.
Mata came to the Supreme Court
and prayed that the search
warrant be declared invalid for its
alleged failure to comply with the
requisites of the Constitution and
the Rules of Court
20TH CENTURY Film Piracy The government's right to In a letter-complaint dated August W/N the judge YES. In the instant case, the lower court lifted the three questioned
FOX FILM v. CA issue search warrants against a 26, 1985, properly lifted the search warrants against the private respondents on the ground that
citizen's papers and effects is petitioner 20th Century Fox Film search warrants he it acted on the application for the issuance of the said search
circumscribed by the Corporation through counsel issued earlier upon warrants and granted it on the misrepresentations of applicant NBI
requirements mandated in the sought the National Bureau of the application of and its witnesses that infringement of copyright or a piracy of a
searches and seizures provision Investigation's (NBI) assistance in the National particular film have been committed.
of the Constitution the conduct of searches and Bureau of
The lower court, therefore, lifted the three (3) questioned search
seizures in connection with the Investigation on
warrants in the absence of probable cause that the private
latter's anti-film piracy campaign. the basis of the
respondents violated P.D. 49. As found out by the court, the NBI
Specifically, the letter-complaint complaint filed by
agents who acted as witnesses did not have personal knowledge of
alleged that certain videotape the petitioner.
the subject matter of their testimony which was the alleged
outlets all over Metro Manila are
commission of the offense by the private respondents. Only the
engaged in the unauthorized sale
petitioner's counsel who was also a witness during the application
and renting out of copyrighted
for the issuance of the search warrants stated that he had personal
films in videotape form which
knowledge that the confiscated tapes owned by the private
constitute a flagrant violation of
respondents were pirated tapes taken from master tapes belonging
PD 49.
to the petitioner. However, the lower court did not give much
Acting on the letter-complaint, the
credence to his testimony in view of the fact that the master tapes
NBI conducted surveillance and
of the allegedly pirated tapes were not shown to the court during
investigation of the outlets
the application.
pinpointed by the petitioner and
subsequently filed three (3) The presentation of the master tapes of the copyrighted films from
applications for search warrants which the pirated films were allegedly copied, was necessary for the
against the video outlets owned by validity of search warrants against those who have in their
the private respondents. possession the pirated films. The petitioner's argument to the effect
that the presentation of the master tapes at the time of application
On September 4, 1985, the lower
may not be necessary as these would be merely evidentiary in
court issued the desired search
nature and not determinative of whether or not a probable cause
warrants. Armed with the search
exists to justify the issuance of the search warrants is not
warrants, the NBI accompanied by
meritorious. The court cannot presume that duplicate or copied
the petitioner's agents, raided the
tapes were necessarily reproduced from master tapes that it owns.
video outlets and seized the items
described therein. An inventory of The application for search warrants was directed against video tape
the items seized was made and left outlets which allegedly were engaged in the unauthorized sale and
with the private respondents. renting out of copyrighted films belonging to the petitioner
pursuant to P.D. 49.
Acting on a motion to lift search
warrants and release seized
properties filed by the private
respondents, the lower court
issued an order dated October 8,
1985, lifting the three (3) search
warrants issued earlier against the
private respondents by the court. 
The lower court denied a motion
for reconsideration filed by the
petitioner in its order dated
January 2, 1986.
NOLASCO v. CRUZ Rebellion; Mayon Some searches may be made Milagros Aguilar-Roque was W/N the search NO. It is at once evident that the Search Warrant authorizes the
PANO Street, QC without a warrant. Thus, arrested together with Cynthia warrant presented seizure of personal properties vaguely described and not
Section 12, Rule 126, Rules of Nolasco by the Constabulary was valid particularized. It is an all-embracing description which includes
Court, explicitly provides: Security Group. Milagrso had been everything conceivable regarding the Communist Party of the
Section 12. Search without wanted as a high ranking officer of W/N with respect Philippines and the National Democratic Front. It does not specify
warrant of person arrested.—A the CPP. The arrest took place at to the arrest of what the subversive books and instructions are; what the manuals
person charged with an 11:30 a.m. of August 6, 1984. At AGUILAR-ROQUE, not otherwise available to the public contain to make them
offense may be searched for noon of the same day, her the subsequent subversive or to enable them to be used for the crime of rebellion.
dangerous weapons or premises were searched and 428 search of her There is absent a definite guideline to the searching team as to what
anything which may be used as documents, a portable typewriter dwelling should items might be lawfully seized thus giving the officers of the law
proof of the commission of the and 2 boxes were seized. Earlier have been made discretion regarding what articles they should seize as, in fact, taken
offense that day, Judge Cruz Paño issued a with a search also were a portable typewriter and 2 wooden boxes. It is thus in
search warrant for rebellion warrant the nature of a general warrant and infringes on the constitutional
against Milagros. On the basis of mandate requiring particular description of the things to be seized.
the documents seized, charges of Search warrants of similar description were considered null and void
subversion and rebellion were filed for being too general.
but the fiscal''s office merely
charged her and Nolasco with NO. Considering that AGUILAR-ROQUE has been charged with
illegal possession of subversive Rebellion, which is a crime against public order; that the warrant for
materials. Milagros asked for her arrest has not been served for a considerable period of time;
suppression of the evidence on the that she was arrested within the general vicinity of her dwelling;
ground that it was illegally and that the search of her dwelling was made within a half hour of
obtained. The search warrant her arrest, we are of the opinion that in her respect, the search at
described the things to be seized No. 239-B Mayon Street, Quezon City, did not need a search
as "Documents, papers and other warrant; this, for possible effective results in the interest of public
records of the CPP, NPA and NDF, order.
xxx".
PEOPLE v. Caucasian; Probable cause has been Accused was searched and W/N the NO. Warrantless search of the personal effects of an accused has
MALMSTEADT Sagada to Baguio; defined as such facts and arrested while transporting warrantless search been declared by this Court as valid, because of existence of
In Flagrante circumstances which could prohibited drugs (hashish). A crime was illegal probable cause, where the smell of marijuana emanated from a
Delicto; lead a reasonable, discreet and was actually being committed by plastic bag owned by the accused, or where the accused was acting
Bulge on his waist prudent man to believe that an the accused and he was caught in suspiciously, and attempted to flee. Aside from the persistent
offense has been committed, flagrante delicto. Thus, the search reports received by the NARCOM that vehicles coming from Sagada
and that the objects sought in made upon his personal effects were transporting marijuana and other prohibited drugs, their
connection with the offense falls squarely under paragraph (1) Commanding Officer also received information that a Caucasian
are in the place sought to be of the foregoing provisions of law, coming from Sagada on that particular day had prohibited drugs in
searched. Warrantless search which allow a warrantless search his possession. Said information was received by the Commanding
of the personal effects of an incident to a lawful arrest. While it Officer of NARCOM the very same morning that accused came down
accused has been declared by is true that the NARCOM officers by bus from Sagada on his way to Baguio City. When NARCOM
this Court as valid, because of were not armed with a search received the information, a few hours before the apprehension of
existence of probable cause, warrant when the search was herein accused, that a Caucasian travelling from Sagada to Baguio
where the smell of marijuana made over the personal effects of City was carrying with him prohibited drugs, there was no time to
emanated from a plastic bag accused, however, under the obtain a search warrant. It must be observed that, at first, the
owned by the accused, or circumstances of the case, there NARCOM officers merely conducted a routine check of the bus
where the accused was acting was sufficient probable cause for (where accused was riding) and the passengers therein, and no
suspiciously and attempted to said officers to believe that extensive search was initially made. It was only when one of the
accused was then and there officers noticed a bulge on the waist of accused, during the course
flee. committing a crime. Probable of the inspection, that accused was required to present his
cause has been defined as such passport. The failure of accused to present his identification papers,
facts and circumstances which when ordered to do so, only managed to arouse the suspicion of the
could lead a reasonable, discreet officer that accused was trying to hide his identity. For is it not a
and prudent man to believe that regular norm for an innocent man, who has nothing to hide from
an offense has been committed, the authorities, to readily present his identification papers when
and that the objects sought in required to do so? The receipt of information by NARCOM that a
connection with the offense are in Caucasian coming from Sagada had prohibited drugs in his
the place sought to be searched. possession, plus the suspicious failure of the accused to produce his
The required probable cause that passport, taken together as a whole, led the NARCOM officers to
will justify a warrantless search reasonably believe that the accused was trying to hide something
and seizure is not determined by illegal from the authorities. From these circumstances arose a
any fixed formula but is resolved probable cause which justified the warrantless search that was
according to the facts of each case. made on the personal effects of the accused.
PEOPLE v. MUSA PLAIN VIEW DOCTRINE OF "PLAIN VIEW”. A civilian informer gave the W/N all the NO. In the case at bar, the NARCOM agents searched the person of
DOCTRINE — The warrantless search and information that Mari Musa was evidence herein the appellant after arresting him in his house but found nothing.
seizure, as an incident to a engaged in selling marijuana in confiscated be They then searched the entire house and, in the kitchen, found and
suspect's lawful arrest, may Suterville, Zamboanga City. Sgt. excluded under seized a plastic bag hanging in a corner. The warrantless search and
extend beyond the person of Ani was ordered by NARCOM the exclusionary seizure, as an incident to a suspect's lawful arrest, may extend
the one arrested to include the leader T/Sgt. Belarga, to conduct a rule beyond the person of the one arrested to include the premises or
premises or surroundings surveillance and test buy on Musa. surroundings under his immediate control. Objects in the "plain
under his immediate control. The civilian informer guided Ani to view" of an officer who has the right to be in the position to have
Objects in the "plain view" of Musa’s house and gave the that view are subject to seizure and may be presented as evidence.
an officer who has the right to description of Musa. Ani was able The " PLAIN VIEW " doctrine may not, however, be used to launch
be in the position to have that to buy one newspaper-wrapped unbridled searches and indiscriminate seizures nor to extend a
view are subject to seizure and dried marijuana for P10.00. general exploratory search made solely to find evidence of
may be presented as evidence. defendant's guilt. The "PLAIN VIEW" DOCTRINE is usually applied
The "plain view" doctrine may The next day, a buy-bust was where a police officer is not searching for evidence against the
not, however, be used to planned. Ani was to raise his right accused, but nonetheless inadvertently comes across an
launch unbridled searches and hand if he successfully buys incriminating object.
indiscriminate seizures nor to marijuana from Musa. As Ani It has also been suggested that even if an object is observed in
extend a general exploratory proceeded to the house, the "plain view," the "plain view" doctrine will not justify the seizure of
search made solely to find NARCOM team positioned the object where the incriminating nature of the object is not
evidence of defendant's guilt. themselves about 90 to 100 apparent from the "plain view" of the object. Stated differently, it
The "plain view" doctrine is meters away. From his position, must be “ IMMEDIATELY APPARENT” to the police that the items
usually applied where a police Belarga could see what was going that they observe may be evidence of a crime, contraband, or
officer is not searching for on. Musa came out of the house otherwise subject to seizure.
evidence against the accused, and asked Ani what he wanted. In the instant case, the appellant was arrested and his person
but, nonetheless inadvertently Ani said he wanted more searched in the living room. Failing to retrieve the marked money
comes across an incriminating marijuana and gave Musa the which they hoped to find, the NARCOM agents searched the whole
object. It has also been P20.00 marked money. Musa went house and found the plastic bag in the kitchen. The plastic bag was,
suggested that even if an into the house and came back, therefore, not within their "plain view" when they arrested the
object is observed in "plain giving Ani two newspaper appellant as to justify its seizure. The NARCOM agents had to move
view," the "plain view" wrappers containing dried from one portion of the house to another before they sighted the
doctrine will not justify the marijuana. Ani opened and plastic bag.
seizure of the object where the inspected it. He raised his right Moreover, when the NARCOM agents saw the plastic bag hanging in
incriminating nature of the hand as a signal to the other one corner of the kitchen, they had no clue as to its contents. They
object is not apparent from the had to ask the appellant what the bag contained. When the
"plain view" of the object. NARCOM agents, and the latter appellant refused to respond, they opened it and found the
Stated differently, it must be moved in and arrested Musa inside marijuana
immediately apparent to the the house. Belarga frisked Musa in
police that the items that they the living room but did not find the
observe may be evidence of a marked money (gave it to his wife
crime, contraband, or who slipped away). T/Sgt. Belarga
otherwise subject to seizure. and Sgt. Lego went to the kitchen
and found a ‘cellophane colored
white and stripe hanging at the
corner of the kitchen.’ They asked
Musa about its contents but failed
to get a response. So they opened
it and found dried marijuana
leaves inside. Musa was then
placed under arrest.
TERRY v. OHIO Stop and Frisk The Fourth Amendment A Cleveland detective (McFadden), Admissibility YES. Where a reasonably prudent officer is warranted in the
applies to "stop and frisk" on a downtown beat which he had against petitioner circumstances of a given case in believing that his safety or that of
procedures such as those been patrolling for many years, of the evidence others is endangered, he may make a reasonable search for
followed here. observed two strangers (petitioner uncovered by the weapons of the person believed by him to be armed and
(a) Whenever a police officer and another man, Chilton) on a search and seizure. dangerous regardless of whether he has probable cause to arrest
accosts an individual and street corner. He saw them that individual for crime or the absolute certainty that the individual
restrains his freedom to walk proceed alternately back and forth is armed.
away, he has "seized" that along an identical route, pausing (a) Though the police must, whenever practicable, secure a warrant
person within the meaning of to stare in the same store window, to make a search and seizure, that procedure cannot be followed
the Fourth Amendment. which they did for a total of about where swift action based upon on-the-spot observations of the
(b) A careful exploration of the 24 times. Each completion of the officer on the beat is required.
outer surfaces of a person's route was followed by a (b) The reasonableness of any particular search and seizure must be
clothing in an attempt to find conference between the two on a assessed in light of the particular circumstances against the
weapons is a "search" under corner, at one of which they were standard of whether a man of reasonable caution is warranted in
that Amendment. joined by a third man (Katz) who believing that the action taken was appropriate.
left swiftly. Suspecting the two (c) The officer here was performing a legitimate function of
men of "casing a job, a stick-up," investigating suspicious conduct when he decided to approach
the officer followed them and saw petitioner and his companions.
them rejoin the third man a couple (d) An officer justified in believing that an individual whose
of blocks away in front of a store. suspicious behavior he is investigating at close range is armed may,
The officer approached the three, to neutralize the threat of physical harm, take necessary measures
identified himself as a policeman, to determine whether that person is carrying a weapon.
and asked their names. The men (e) A search for weapons in the absence of probable cause to arrest
"mumbled something," must be strictly circumscribed by the exigencies of the situation.
whereupon McFadden spun
petitioner around, patted down his
outside clothing, and found in his
overcoat pocket, but was unable
to remove, a pistol. The officer
ordered the three into the store.
He removed petitioner's overcoat,
took out a revolver, and ordered
the three to face the wall with
their hands raised. He patted
down the outer clothing of Chilton
and Katz and seized a revolver
from Chilton's outside overcoat
pocket. He did not put his hands
under the outer garments of Katz
(since he discovered nothing in his
pat-down which might have been
a weapon), or under petitioner's or
Chilton's outer garments until he
felt the guns. The three were
taken to the police station.
Petitioner and Chilton were
charged with carrying concealed
weapons. The defense moved to
suppress the weapons. Though the
trial court rejected the prosecution
theory that the guns had been
seized during a search incident to a
lawful arrest, the court denied the
motion to suppress and admitted
the weapons into evidence on the
ground that the officer had cause
to believe that petitioner and
Chilton were acting suspiciously,
that their interrogation was
warranted, and that the officer, for
his own protection, had the right
to pat down their outer clothing
having reasonable cause to believe
that they might be armed.
PEOPLE v. Latong; Torch; Search incidental to lawful Accused-appellant W/N the trial NO. Under the circumstances obtaining in this case, however,
SOLAYAO Hidden firearms arrest Nilo Solayao was charged with the court erred in accused-appellant's arguments are hardly tenable. He and his
crime of illegal possession of admitting the companions' drunken actuations aroused the suspicion of SPO3
firearm and ammunition defined subject firearm in Niño's group. After SPO3 Niño told accused-appellant not to run
and penalized under Presidential evidence as it was away, the former identified himself as a government agent. The
Decree No. 1866. a product of peace officers did not know that he had committed, or was actually
unlawful committing, the offense of illegal possession of firearm. Tasked with
The lone prosecution witness, warrantless search verifying the report that there were armed men roaming in the
SPO3 Jose Niño, narrated that at barangays surrounding Caibiran, their attention was understandably
about 9PM of July 9, 1992, with drawn to the group that had aroused their suspicion. They could not
CAFGU members Teofilo Llorad, Jr. have known that the object wrapped in coconut leaves which
and Cecilio Cenining, he went to accused-appellant was carrying hid a firearm. The case at bar
Barangay Caulangohan, Caibiran, constitutes an instance where a search and seizure may be effected
Biliran. They were to conduct an without first making an arrest. There was justifiable cause to "stop
intelligence patrol as required of and frisk" accused-appellant when his companions fled upon seeing
them by their intelligence officer the government agents. Under the circumstances, the government
to verify reports on the presence agents could not possibly have procured a search warrant first.
of armed persons roaming around Thus, there was no violation of the constitutional guarantee against
the barangays of Caibiran. unreasonable searches and seizures. Nor was there error on the
part of the trial court when it admitted the homemade firearm as
From Barangay Caulangohan, the
evidence.
team of Police Officer Niño
proceeded to Barangay Onion
In the case at bar, the prosecution was only able to prove by
where they met the group of
testimonial evidence that accused-appellant admitted before Police
accused-appellant
Officer Niño at the time that he was accosted that he did not have
Nilo Solayao numbering five. The
any authority or license to carry the subject firearm when he was
former became suspicious when
asked if he had one. In other words, the prosecution relied on
they observed that the latter were
accused-appellant's admission to prove the second element. . . . By
drunk and that accused-appellant
its very nature, an "admission is the mere acknowledgment of a fact
himself was wearing a camouflage
or of circumstances from which guilt may be inferred tending to
uniform or a jungle suit. Accused-
incriminate the speaker, but not sufficient of itself to establish his
appellant's companions, upon
guilt." In other words, it is a "statement by defendant of fact or facts
seeing the government agents,
pertinent to issues pending, in connection with proof of other facts
fled.
or circumstances, to prove guilt, but which is, of itself, insufficient to
Police Officer Niño told accused- authorize conviction." From the above principles, this Court can
appellant not to run away and infer that an admission in criminal cases is insufficient to prove
introduced himself as "PC," after beyond reasonable doubt the commission of the crime charged.
which he seized the dried coconut Moreover, said admission is extra-judicial in nature. As such, it does
leaves which the latter was not fall under Section 4 of Rule 129 of the Revised Rules of Court . . .
carrying and found wrapped in it a . Not being a judicial admission, said statement by accused-
49-inch long homemade firearm appellant does not prove beyond reasonable doubt the second
locally known as "latong." When element of illegal possession of firearm. It does not even establish
he asked accused-appellant who a  prima facie case. It merely bolsters the case for the prosecution
issued him a license to carry said but does not stand as proof of the fact of absence or lack of a
firearm or whether he was license.
connected with the military or any
intelligence group, the latter
answered that he had no
permission to possess the same.
Thereupon, SPO3 Niño confiscated
the firearm and turned him over
to the custody of the policemen of
Caibiran who subsequently
investigated him and charged him
with illegal possession of firearm.
Accused-appellant, in his defense,
did not contest the confiscation of
the shotgun but averred that this
was only given to him by one of his
companions, Hermogenes
Cenining, when it was still
wrapped in coconut leaves. He
claimed that he was not aware
that there was a shotgun
concealed inside the coconut
leaves since they were using the
coconut leaves as a torch. He
further claimed that this was the
third torch handed to him after the
others had been used up. This was
corroborated by one Pedro Balano.
MALACAT v. CA Eyes moving very In a search incidental to a Ombudsman Vasquez required W/N the NO. The general rule as regards arrests, searches and seizures is
fast lawful arrest, as the precedent Rogado and Rivera of Economic warrantless arrest that a warrant is needed in order to validly effect the same. The
arrest determines the validity Intelligence and Investigation of petitioner is Constitutional prohibition against unreasonable arrests, searches
of the incidental search, the Bureau (EIIB) to produce all valid. and seizures refers to those effected without a validly issued
legality of the arrest is documents relating to Personal warrant, subject to certain exceptions. As regards valid warrantless
questioned in a large majority Service Funds yr. 1988 and all arrests, these are found in Section 5, Rule 113 of the Rules of Court,
of these cases, e.g., whether evidence for the whole plantilla of which reads, in part: Sec. 5. Arrest, without warrant; when lawful A
an arrest was merely used as a EIIB for 1988. The subpoena duces peace officer or a private person may, without a warrant, arrest a
pretext for conducting a tecum was issued in connection person:
search. In this instance, the law with the investigation of funds (a) When, in his presence, the person to be arrested has committed,
requires that there first be a representing savings from unfilled is actually committing, or is attempting to commit an offense;
lawful arrest before a search positions in the EIIB which were (b) When an offense has in fact just been committed, and he has
can be made — the process legally disbursed. Almonte and personal knowledge of facts indicating that the person to be
cannot be reversed. At bottom, Perez denied the anomalous arrested has committed it; and
assuming a valid arrest, the activities that circulate around the (c) When the person to be arrested is a prisoner who has escaped
arresting officer may search EIIB office. They moved to quash
the person of the arrestee and the subpoena duces tecum. They In the instant petition, the trial court validated the warrantless
the area within which the claim privilege of an agency of the search as a "stop and frisk" with "the seizure of the grenade from
latter may reach for a weapon Government. the accused as an appropriate incident to his arrest," hence
or for evidence to destroy, and necessitating a brief discussion on the nature of these exceptions to
seize any money or property Petitioner Jose T. Almonte was the warrant requirement. At the outset, we note that the trial court
found which was used in the formerly Commissioner of the EIIB, confused the concepts of a "STOP-AND-FRISK" and of a SEARCH
commission of the crime, or while Villamor C. Perez is Chief of INCIDENTAL TO A LAWFUL ARREST. These two types of warrantless
the fruit of the crime, or that the EIIB's Budget and Fiscal searches differ in terms of the requisite quantum of proof before
which may be used as Management Division. The they may be validly effected and in their allowable scope.
evidence, or which might subpoena duces tecum was issued In a SEARCH INCIDENTAL TO A LAWFUL ARREST, as the precedent
furnish the arrestee with the by the Ombudsman in connection arrest determines the validity of the incidental search, the legality of
means of escaping or with his investigation of an the arrest is questioned in a large majority of these cases, e.g.,
committing violence anonymous letter alleging that whether an arrest was merely used as a pretext for conducting a
funds representing savings from search.
unfilled positions in the EIIB had In this instance, the law requires that there first be a lawful arrest
been illegally disbursed. The letter, before a search can be made the process cannot be reversed. At
purporting to have been written by bottom, assuming a valid arrest, the arresting officer may search the
an employee of the EIIB and a person of the arrestee and the area within which the latter may
concerned citizen, was addressed reach for a weapon or for evidence to destroy, and seize any money
to the Secretary of Finance, with or property found which was used in the commission of the crime,
copies furnished several or the fruit of the crime, or that which may be used as evidence, or
which might furnish the arrestee with the means of escaping or
government offices, including the committing violence.
Office of the Ombudsman. Here, there could have been no valid in flagrante delicto or hot
pursuit arrest preceding the search in light of the lack of personal
knowledge on the part of Yu, the arresting officer, or an overt
physical act, on the part of petitioner, indicating that a crime had
just been committed, was being committed or was going to be
committed.
PAPA v. MAGO Tariff and Custom The Tariff and Customs Code Martin Alagao, head of the W/N petitioners YES. In the instant case, we note that petitioner Martin Alagao and
Code does not require a search counter- intelligence unit of the were allowed to his companion policemen did not have to make any search before
warrant for purposes of Manila Police Department, acting search and seize they seized the two trucks and their cargo. In their original petition,
enforcing customs and tariff upon a reliable information the articles even and amended petition, in the court below Remedios Mago and
laws. Under Sec. 2203 thereof, received on 3 November 1966 to without a warrant Valentin Lanopa did not even allege that there was a search. All that
persons having police authority the effect that a certain shipment they complained of was,
may enter, pass through or of personal effects, allegedly
search any land, enclosure, misdeclared and undervalued, "That while the trucks were on their way, they were intercepted
warehouse, store or building would be released the following without any search warrant near the Agrifina Circle and taken to the
not being a dwelling house and day from the customs zone of the Manila Police, where they were detained." But even if there was a
also, to inspect, search and port of Manila and loaded on two search, there is still authority to the effect that no search warrant
examine any vehicle or aircraft trucks, and upon orders of Ricardo would be needed under the circumstances obtaining in the instant
and any trunk, package, box or Papa, Chief of Police of Manila and case. The guaranty of freedom from unreasonable searches and
envelope or any person on a duly deputized agent of the seizures is construed as recognizing a necessary difference
board or stop and search and Bureau of Customs, conducted between a search of a dwelling house or other structure in respect
examine any vehicle, beast or surveillance at gate 1 of the of which a search warrant may readily be obtained and a search of
person suspected of holding or customs zone. When the trucks a ship, motorboat, wagon, or automobile for contraband goods ,
conveying any dutiable or left gate 1 at about 4:30 p.m. of 4 where it is not practicable to secure a warrant , because the
prohibited article introduced November 1966, elements of the vehicle can be quickly moved out of the locality or jurisdiction in
into the Philippines contrary to counter-intelligence unit went which the warrant must be sought.
law, without mentioning the after the trucks and intercepted
need of a search warrant in them at the Agrifina Circle, Ermita,
said cases. Except in the search Manila. The load of the two trucks,
of a dwelling house, therefore, consisting of nine bales of goods,
persons exercising police and the two trucks, were seized on
authority under the customs instructions of the Chief of Police.
law may effect search and Upon investigation, a person
seizure without search warrant claimed ownership of the goods
in the enforcement of customs and showed to the policemen a
laws. "Statement and Receipts of Duties
Collected on Informal Entry No.
147-5501", issued by the Bureau of
Customs in the name of a certain
Bienvenido Naguit. Claiming to
have been prejudiced by the
seizure and detention of the two
trucks and their cargo, Remedios
Mago and Valentin B. Lanopa filed
with the Court of First Instance
(CFI) of Manila a petition "for
mandamus with restraining order
or preliminary injunction (Civil
Case 67496), praying for the
issuance of a restraining order, ex
parte, enjoining the police and
customs authorities, or their
agents, from opening the bales
and examining the goods, and a
writ of mandamus for the return of
the goods and the trucks, as well
as a judgment for actual, moral
and exemplary damages in their
favor.
PEOPLE v. Search while On or about March 7, 1999, in W/N the search in YES. The court held that the police had probable cause to effect the
TUAZON driving Antipolo City, Bernardo Tuazon the car was valid warrantless search of the Gemini car driven by appellant. A
have in his possession seven (7) and the articles confidential informer tipped them off that said car was going to
heat-sealed transparent plastic were admissible to deliver shabu at Marville Subdivision. Pursuing said lead,
bags of methylamphetamine evidence the Antipolo City police sent a team to Marville Subdivision to
hydrochloride also known as shabu monitor said vehicle. The information provided by the informer
for a total weight of 250.74 grams. turned out to be correct as, indeed, the Gemini car was spotted in
Upon arraignment, appellant, the place where it was said to be bringing shabu. When they
pleaded not guilty. stopped the car, they saw a gun tucked in appellant’s
The prosecutions waist. Appellant did not have any document to support his
version of the case relied heavily possession of said firearm which all the more strengthened the
on the testimony of PO3 Glenon police officers’ suspicion. After he was told to step out of the car,
Bueno. In the Joint Affidavit they found on the driver’s seat plastic sachets containing white
executed by PO3 Bueno and PO1 powdery substance. These circumstances, taken together, are
Padlan, it was stated that when sufficient to establish probable cause for the warrantless search of
they frisked appellant, they the Gemini car.
discovered 2 big plastic bags and 5
medium size plastics and a 9 mm.
pistol marked Parabellum bearing
serial number C-9890 with one
loaded magazine with eleven
ammunition.
The trial court found the
evidence presented by the
prosecution sufficient to support a
guilty verdict and imposed upon
appellant the penalty of reclusion
perpetua and to pay a fine
of P500,000.00. On September 17,
2003, the Court resolved to accept
the appeal interposed by appellant
and required the parties to file
their respective briefs. In addition
to the required brief, appellant
filed a supplementary pleading in
which he questioned the validity of
his arrest and the admissibility of
the evidence presented against
him. He contends that at the time
of his warrantless arrest, he was
merely driving within Marville
Subdivision. He had not
committed, was not committing,
and was not about to commit any
crime which could have justified
his apprehension. He goes on to
argue that even if he had waived
the issue regarding the validity of
his arrest by his failure to raise the
matter before entering his plea,
such waiver did not affect the
unlawfulness of the search and
seizure conducted by the
police. Appellant claims that as the
confidential informant had been
cooperating with the police for
three weeks prior to his arrest, the
authorities were already informed
of his identity and his alleged
illegal activities. They should have
conducted a prior surveillance and
then sought a search warrant from
the court. Absent said warrant,
the shabu seized from him should
be excluded from evidence. The
Court of Appeals affirmed the
findings and conclusion of the
court a quo.
PEOPLE v. DE Eurocar Sales Search Warrant - It is an order In the early morning of December W/N there was a YES. Raiding Military Operatives, At The Height Of The December
GRACIA Building in writing issued in the name of 1, 1989, Maj. Efren Soria of the valid search 1989 Coup D’etat Attempt, Had Reasonable Ground To Believe That
the People of the Philippines, Intelligence Division, National warrant A Crime Was Being Committed, And Had No Opportunity To Apply
signed by a judge and directed Capital Region Defense Command, For And Secure A Search Warrant From The Courts, The Same
to a peace officer, was conducting a surveillance of Constituted An Exception To The Prohibition Against Warrantless
commanding him to search for the Eurocar Sales Office located at Searches.
personal property described Epifanio de los Santos Avenue in
therein and to bring it before Quezon City, together with his It is admitted that the military operatives who raided the Eurocar
the court team. The surveillance, which Sales Office were not armed with a search warrant at that time. The
actually started on the night of raid was actually precipitated by intelligence reports that said office
November 30, 1989 at around was being used as headquarters by the RAM. Prior to the raid, there
10:00 P.M., was conducted was a surveillance conducted on the premises wherein the
pursuant to an intelligence report surveillance team was fired at by a group of men coming from the
received by the division that said Eurocar building. When the military operatives raided the place, the
establishment was being occupied occupants thereof refused to open the door despite requests for
by elements of the RAM-SFP as a them to do so, thereby compelling the former to break into the
communication command post. office.
Sgt. Crispin Sagario, the driver of The Eurocar Sales Office is obviously not a gun store and it is
the car, parked the vehicle around definitely not an armory or arsenal which are the usual depositories
ten to fifteen meters away from for explosives and ammunition. It is primarily and solely engaged in
the Eurocar building, had earlier the sale of automobiles. The presence of an unusual quantity of
alighted from the car to conduct high-powered firearms and explosives could not be justifiably or
his surveillance on foot. even colorably explained.
A crowd was then gathered near
the Eurocar office watching the In addition, there was general chaos and disorder at that time
on-going bombardment near because of simultaneous and intense firing within the vicinity of the
Camp Aguinaldo. After a while, a office and in the nearby Camp Aguinaldo which was under attack by
group of five men disengaged rebel forces. The courts in the surrounding areas were obviously
themselves from the crowd and closed and, for that matter, the building and houses therein were
walked towards the car of the deserted.
surveillance team. At that
moment, Maj. Soria, who was then
seated in front, saw the
approaching group and
immediately ordered Sgt. Sagario
to start the car and leave the area.
As they passed by the group, then
only six meters away, the latter
pointed to them, drew their guns
and fired at the team, which attack
resulted in the wounding of Sgt.
Sagario on the right thigh. Nobody
in the surveillance team was able
to retaliate because they sought
cover inside the car and they were
afraid that civilians or bystanders
might be caught in the cross-fire.
ANIAG v. Gun ban and The COMELEC issued two W/N warrantless NO. In the case at bench, we find that the checkpoint was set up
COMELEC checkpoint; 1992 resolutions in preparation for the search was valid twenty (20) meters from the entrance to the Batasan Complex to
Elections; Driver synchronized 1992 national and enforce Resolution No. 2327. 
local elections.
The first resolution is Resolution No. There was no evidence to show that the policemen were impelled
2323 otherwise referred to as the to do so because of a confidential report leading them to reasonably
"Gun Ban. believe that certain motorists matching the description furnished by
The second resolution Resolution their informant were engaged in gunrunning, transporting firearms
No. 2327 providing for the summary or in organizing special strike forces. Nor, as adverted to earlier, was
disqualification of candidates there any indication from the package or behavior of Arellano that
engaged in gunrunning, using and could have triggered the suspicion of the policemen. Absent such
transporting of firearms, organizing justifying circumstances specifically pointing to the culpability of
special strike forces, and petitioner and Arellano, the search could not be valid. 
establishing spot checkpoints. It may be argued that the seeming acquiescence of Arellano to the
Pursuant to the "Gun Ban," Mr. search constitutes an implied waiver of petitioner's right to question
Serapio P. Taccad, Sergeant-at- the reasonableness of the search of the vehicle and the seizure of
Arms, House of Representatives, the firearms.
wrote petitioner who was then a In the case of the petitioner, only his driver was in the car at that
Congressman of the 1st District of time it was stopped for inspection. As conceded by COMELEC, driver
Bulacan requesting the return of the Arellano did not know the purpose of the checkpoint. In the face of
two (2) firearms3 issued to him by fourteen (14) armed policemen conducting the operation, driver
the House of Representatives.  Arellano being alone and a mere employee of the petitioner could
Upon being advised of the request not have marshaled the strength and the courage to protest against
on 13 January 1992 by his staff, the extensive search conducted in the vehicle. In such scenario, the
petitioner immediately instructed "implied acquiescence," if there was any, could not be more than a
his driver, Ernesto Arellano, to pick mere passive conformity on Arellano's part to the search, and
up the firearms from petitioner's "consent" given under intimidating or coercive circumstances is no
house at Valle Verde and return consent within the purview of the constitutional guaranty.
them to Congress. About thirty
minutes later, the policemen
manning the outpost flagged down
the car driven by Arellano as it
approached the checkpoint. They
searched the car and found the
firearms neatly packed in their gun
cases and placed in a bag in the
trunk of the car. Arellano was then
apprehended and detained. He
explained that he was ordered by
petitioner to get the firearms from
the house and return them to
Sergeant-at-Arms Taccad of the
House of Representatives.
The police referred Arellano's case
to the Office of the City Prosecutor
for an inquest.
The City Prosecutor ordered
the release of Arellano after finding
the latter's sworn explanation
meritorious.
The City Prosecutor invited
petitioner to shed light on the
circumstances mentioned in
Arellano's sworn explanation.
Petitioner not only appeared at the
preliminary investigation to confirm
Arellano's statement but also wrote
the City Prosecutor urging him to
exonerate Arellano. He explained
that Arellano did not violate the
firearms ban as he, in fact, was
complying with it when
apprehended by returning the
firearms to Congress; and, that he
was petitioner's driver, not a
security officer nor a bodyguard
ESPANO v. CA Marijuana in An exception to the said rule is : The evidence for the prosecution, W/N the NO. In this case, the ten cellophane bags of marijuana seized at
house a warrantless search incidental based on the testimony of Pat. marijuana petitioner's house after his arrest at Pandacan and Zamora Streets
to a lawful arrest for Romeo Pagilagan, shows that on confiscated from do not fall under the said exceptions. 
dangerous weapons or July 14, 1991, at about 12:30 a.m., the house in As regards the brick of marijuana found inside the appellant's
anything which may be used as he and other police officers, addition to the house, the trial court correctly ignored it apparently in view of its
proof of the commission of an namely, Pat. Wilfredo Aquilino, marijuana inadmissibility. While initially the arrest as well as the body search
offense. It may extend beyond Simplicio Rivera, and Erlindo confiscated while was lawful, the warrantless search made inside the appellant's
the person of the one arrested Lumboy of the Western Police Espano waa house became unlawful since the police operatives were not armed
to include the premises or District (WPD), Narcotics Division frisked may be with a search warrant. Such search cannot fall under "SEARCH
surroundings under his went to Zamora and Pandacan used as evidence MADE INCIDENTAL TO A LAWFUL ARREST," the same being limited
immediate control Streets, Manila to confirm reports to body search and to that point within reach or control of the
of drug pushing in the area. They person arrested, or that which may furnish him with the means of
saw petitioner selling "something" committing violence or of escaping. In the case at bar, appellant was
to another person. After the admittedly outside his house when he was arrested. Hence, it can
alleged buyer left, they hardly be said that the inner portion of his house was within his
approached petitioner, identified reach or control. 
themselves as policemen, and The warrantless search made in his house, however, which yielded
frisked him. The search yielded ten cellophane bags of marijuana became unlawful since the police
two plastic cellophane tea bags of officers were not armed with a search warrant at the time.
marijuana . When asked if he had Moreover, it was beyond the reach and control of petitioner.
more marijuana, he replied that
there was more in his house. The
policemen went to his residence
where they found ten more
cellophane tea bags of marijuana.
Petitioner was brought to the
police headquarters where he was
charged with possession of
prohibited drugs. On July 24, 1991,
petitioner posted bail 3 and the
trial court issued his order of
release on July 29, 1991. 
PEOPLE v. DEL Buy-bust fail A search warrant is not a Upon application of SPO3 W/N the NO. According to the version of the prosecution, during the alleged
ROSARIO sweeping authority Raymundo Untiveros, RTC Judge implementation of buybust operation, accused-appellant handed over to Veneracion
empowering a raiding party to Arturo de Guia issued in the the search warrant Luna, the alleged poseur- buyer, a quantity of shabu, and Luna in
undertake a finishing morning of September 4, 1991 a was lawful and turn paid accused-appellant a marked 100 bill and then returned to
expedition to seize and search warrant authorizing the that the object the police station and informed the raiding team that he had
confiscate any and all kinds of search and seizure of an seized may be already bought the shabu from accused-appellant. Thereupon, the
evidence or articles relating to "undetermined quantity of used to prove Del raiding team proceeded to the house of accused- appellant to
a crime. The Constitution itself Methamphetamine Hydrochloride Rosario’s guilt implement the search warrant. The version of the prosecution is
and the Rules of Court, commonly known as shabu and its highly incredible. The record is devoid of any reason why the police
specifically mandate that the paraphernalias" in the premises of W/N the officers did not make any attempt to arrest accused- appellant at
search warrant must appellant's house. However, the ammunition was the time he allegedly sold the shabu to Veneracion Luna who was
particularly describe the things search warrant was not validly seized as an accompanied by another police officer. That was the opportune
to be seized. Thus, the search implemented immediately due to incident to a lawful moment to arrest accused-appellant. The version foisted by the
warrant was no authority for the lack of police personnel to arrest prosecution upon this Court is contrary to human experience in the
the police officers to seize the form the raiding team. At about 9 ordinary course of human conduct. The usual procedure in a buy-
firearm which was not o'clock in the evening of that day, bust operation is for the police officers to arrest the pusher of drugs
mentioned, much less a raiding team was finally at the very moment he hands over the dangerous drug to the
described with particularity, in organized. In the final briefing of poseur-buyer. That is the every reason why such a police operation
the search warrant . the raiding team at the police is called a "BUY-BUST" operation. The police poseur-buyer "buys”
station, it was agreed upon that dangerous drugs from the pusher and "bust" (arrests) him the
PO1 Venerando Luna will buy moment the pusher hands over the drug to the police officer.
shabu from appellant and after his Neither may it be maintained that the gun was seized in the course
return from appellant's house, the of an arrest, for as earlier observed, Del Rosario's arrest was far
raiding team will implement the from regular and legal. Said firearm, having been illegally seized, the
search warrant. A marked money same is not admissible in evidence.
consisting of a P100 bill bearing
serial no. PQ 329406 was given by
the Station Commander to PO1
Luna and entered in the police
logbook PO1 Luna with a
companion proceeded to
appellant's house to implement
the search warrant. Barangay
Capt. Maigue, Norma del Rosario
and appellant witnessed the
search at appellant's house. SPO3
de la Cruz and PO3 Francisco
found a black canister containing
shabu, an aluminum foil, a paltik .
22 caliber atop the TV set, three
used ammunitions in a cup and
three wallets, one containing the
marked money. SPO1 Novero
found inside a show box aluminum
foils, napkins and a burner.
Normando del Rosario was
charged with Illegal Possession of
Firearm and Ammunitions and
Illegal Sale of Regulated Drugs.
UMIL v. RAMOS Ronnie Javelon The absence of a judicial The Regional Intelligence W/N Dural can be YES. Dural was arrested for being a member of the New Peoples
aka Rolando warrant is no legal impediment Operations Unit of the Capital validly arrested Army (NPA), an outlawed subversive organization. Subversion being
Dural ; Murder to arresting or capturing Command (RIOU-CAPCOM) without any a continuing offense, the arrest of Rolando Dural without warrant is
CAPCOM persons committing overt acts received confidential information warrant of arrest justified as it can be said that he was committing an offense when
of violence against government about a member of the NPA for the crime of arrested. The crimes of rebellion, subversion, conspiracy or proposal
forces, or any other milder acts Sparrow Unit (liquidation squad) rebellion to commit such crimes, and crimes or offenses committed in
but equally in pursuance of the being treated for a gunshot wound furtherance thereof or in connection therewith constitute direct
rebellious movement. The at the St. Agnes Hospital in assaults against the State and are in the nature of continuing
arrest or capture is thus Roosevelt Avenue, Quezon City. crimes. The arrest of persons involved in the rebellion whether as its
impelled by the exigencies of Upon verification, it was found fighting armed elements, or for committing non-violent acts but in
the situation that involves the that the wounded person, who furtherance of the rebellion, is more an act of capturing them in the
very survival of society and its was listed in the hospital records course of an armed conflict, to quell the rebellion, than for the
government and duly as Ronnie Javelon, is actually purpose of immediately prosecuting them in court for a statutory
constituted authorities. Rolando Dural, a member of the offense. The arrest, therefore, need not follow the usual procedure
NPA liquidation squad, responsible in the prosecution of offenses which requires the determination by a
for the killing of 2 CAPCOM judge of the existence of probable cause before the issuance of a
soldiers the day before. Dural was judicial warrant of arrest and the granting of bail if the offense is
then transferred to the Regional bailable
Medical Services of the CAPCOM,
for security reasons. While
confined thereat, Dural was
positively identified by
eyewitnesses as the gunman who
went on top of the hood of the
CAPCOM mobile patrol car, and
fired at the 2 CAPCOM soldiers
seated inside the
car.Consequently, Dural was
referred to the Caloocan City Fiscal
who conducted an inquest and
thereafter filed with the Regional
Trial Court of Caloocan City an
information charging Rolando
Dural alias Ronnie Javelon with the
crime of ―Double Murder with
Assault Upon Agents of Persons in
Authority.
A petition for habeas corpus was
filed with the Supreme Court on
behalf of Roberto Umil, Rolando
Dural, and Renato Villanueva. The
Court issued the writ of habeas
corpus. A Return of the Writ was
filed. Umil and Villanueva posted
bail before the Regional Trial Court
of Pasay City where charges for
violation of the Anti-Subversion
Act had been filed against them,
and they were accordingly
released. 
WEBB v. DE LEON Vizconde
Massacre
PEOPLE v. SUCRO Church Search and seizures supported Pat. Fulgencio went to Arlie W/N the arrest YES. It was held that when a police officer sees the offense, although
by a valid warrant of arrest is Regalado‘s house at C. Quimpo to without warrant is at a distance, or hears the distrurbances created thereby, and
not an absolute rule. Rule 126, monitor activities of Edison SUCRO lawful.  proceeds at once to the scene thereof, he may effect an arrest
Sec 12 of Rules of Criminal (accused). Sucro was reported to without a warrant. The offense is deemed committed in the
Procedure provides that a be selling marijuana at a chapel 2 presence or within the view of the officer.
person lawfully arrested may meters away from Regalado’s
be searched for dangerous house. Sucro was monitored to
weapons or anything, which have talked and exchanged things
may be used as proof of the three times. These activities are
commission of an offense, reported through radio to P/Lt.
without a search warrant Seraspi. A third buyer was
transacting with appellant and was
However, warantless search reported and later identified as
and seizures are legal as long Ronnie Macabante. From that
as PROBABLE CAUSE existed. moment, P/Lt.Seraspi proceeded
The police officers have to the area. While the police
personal knowledge of the officers were at the Youth Hostel
actual commission of the crime in Maagama St. Fulgencio told Lt.
from the surveillance of the Seraspi to intercept. Macabante
activities of the accused. As was intercepted at Mabini and
police officers were the ones Maagama crossing in front of
conducting the surveillance, it Aklan Medical center. Macabante
is presumed that they are saw the police and threw a tea bag
regularly in performance of of marijuana on the ground.
their duties.  Macabante admitted buying the
marijuana from Sucro in front of
the chapel. 
The police team intercepted and
arrested SUCRO at the corner of C.
Quimpo and Veterans. Recovered
were 19 sticks and 4 teabags of
marijuana from a cart inside the
chapel and another teabag from
Macabante. 
PEOPLE v. Buybust; father A buy-bust operation is a form CIC Taduran together with S/Sgt. W/N the evidence NO. In the instant case, however, the procedure adopted by the
RODRIGUEZA and son of entrapment employed by Molinawe and other officers confiscated during NARCOM agents failed to meet this qualification. Based on the very
peace officers to trap and received from a confidential the raid conducted evidence of the prosecution, after the alleged consummation of the
catch a malefactor in flagrante informer that there was an in the house of sale of dried marijuana leaves, CIC Taduran immediately released
delicto. Applied to the case at ongoing illegal traffic of prohibited Jovencio appellant Rodrigueza instead of arresting and taking him into his
bar, the term in flagrante drugs in Tagas, Daraga, Albay. Sgt. Rodrigueza is custody. This act of CIC Taduran, assuming arguendo that the
delicto requires that the Molinawe gave the money to admissible in supposed sale of marijuana did take place, is decidedly contrary to
suspected drug dealer must be Taduran who acted as the poseur evidence.  the natural course of things and inconsistent with the aforestated
caught redhanded in the act of buyer. He was told to look for a purpose of a buy-bust operation. It is rather absurd on his part to
selling marijuana or any certain Don, the alleged seller of let appellant escape without having been subjected to the
prohibited drug to a person prohibited drugs. After agreeing on sanctions imposed by law. It is, in fact, a dereliction of duty by an
acting or posing as a buyer.  the price of P200.00 for 100 grams agent of the law. The raid conducted by the NARCOM agents in
of marijuana, Don halted and later the house of Jovencio Rodrigueza was not authorized by any
on Don gave Taduran "a certain search warrant. It does not appear, either, that the situation falls
object wrapped in a plastic" which under any of the aforementioned cases. Hence, appellant's right
was later identified as marijuana, against unreasonable search and seizure was clearly violated. The
and received payment therefor. NARCOM agents could NOT have justified their act by invoking the
Thereafter, Taduran returned to urgency and necessity of the situation because the testimonies of
the headquarters and made a the prosecution witnesses reveal that the place had already been
report regarding his said purchase put under surveillance for quite some time. Had it been their
of marijuana. Subsequently, Major intention to conduct the raid, then they should, because they
Zeidem ordered a team to conduct easily could, have first secured a search warrant during that time. 
an operation to apprehend the (In the case at bar, the police officer, acting as poseur-buyer in a
suspects. In the evening of the ―buy-bust operation‖, inst5ead of arresting the suspect and taking
same date, appellant, Lonceras him into custody after the sale, returned to police headquarters and
and Segovia was arrested. The filed his report. It was only in the evening of the same day that the
constables were not, however, police officer, without a warrant, arrested the suspect at the latter‘s
armed with a warrant of arrest house where dried marijuana leaves were found and confiscated. It
when they apprehended the three was held that the arrest and the seizure were unlawful.)
accused. 
Thereafter, agents of the Narcotics
Command (NARCOM) conducted a
raid in the house of Jovencio
Rodrigueza, father of appellant.
During the raid, they were able to
confiscate dried marijuana leaves
and a plastic syringe, among
others. The search, however, was
not authorized by any search
warrant. The RTC found
Rodrigueza guilty of violating the
Dangerous Drug Act. 
GO v. CA Rolito Go v. Eldon Section 5 of Rule 113 of the On July 2, 1991, petitioner entered W/N a lawful NO. In this case, there was no lawful warrantless arrest of petitioner
Maguan 1985 Rules on Criminal Wilson St., where it is a one-way warrantless arrest within the meaning of Section 5 of Rule 113. Petitioner's "arrest"
Procedure provides that ―a street and started travelling in the had been effected took place six (6) days after the shooting of Maguan. The
peace officer or a private opposite or "wrong" direction. At by the San Juan "arresting" officers obviously were not present, within the meaning
person may, without warrant, the corner of Wilson and J. Abad Police in respect of of Section 5(a), at the time petitioner had allegedly shot Maguan.
arrest a person‖:  Santos Sts., petitioner's and Eldon petitioner Go.  Neither could the "arrest" effected six (6) days after the shooting
(a) When, in his presence, the Maguan's cars nearly bumped be reasonably regarded as effected "when [the shooting had] in
person to be arrested has each other. Petitioner alighted fact just been committed" within the meaning of Section 5(b).
committed, is actually from his car, walked over and shot Moreover, none of the "arresting" officers had any "personal
committing, or is attempting to Maguan inside his car. Petitioner knowledge" of facts indicating that petitioner was the gunman
commit an offense; then boarded his car and left the who had shot Maguan. The information upon which the police
(b) When an offense has in fact scene. A security guard at a nearby acted had been derived from statements made by alleged
just been committed, and he restaurant was able to take down eyewitnesses to the shooting. That information did not, however,
has personal knowledge of petitioner's car plate number. constitute "personal knowledge"
facts indicating that the person Verification at the LTO showed
to be arrested has committed that the car was registered to one
it Elsa Ang Go.The security guard of
(c) escaped prisoner the bake shop positively identified
Go as the same person who had
shot Maguan. 
The police launched a manhunt for
petitioner. On July 8, 1991,
Petitioner presented himself
before the San Juan Police Station
to verify news reports that he was
being hunted by the police; he was
accompanied by two (2) lawyers.
The police forthwith detained him.
That same day, the police
promptly filed a complaint for
frustrated homicide against
petitioner with the Office of the
Provincial Prosecutor of Rizal. The
Prosecutor filed an information for
murder before the RTC. Counsel
for petitioner filed with the
Prosecutor an omnibus motion for
immediate release and proper
preliminary investigation, alleging
that the warrantless arrest of
petitioner was unlawful and that
no preliminary investigation had
been conducted before the
information was filed.

POSADAS v. CA Buri Bag As between a warrantless On October 16, 1986, Patrolman W/N the YES. At the time the peace officers in this case identified themselves
search and seizure conducted Ursicio Ungab and Pat. Umbra warrantless search and apprehended the petitioner as he attempted to flee they did not
at military or police Umpar, both members of the on the person of know that he had committed, or was actually committing the
checkpoints and the search Integrated National Police (INP) of petitioner is valid.  offense of illegal possession of firearms and ammunitions. They just
thereat in the case at bar, Davao assigned with the suspected that he was hiding something in the buri bag. The said
there is no question that, Intelligence Task Force, were circumstances did not justify an arrest without a warrant. 
indeed, the latter is more conducting a surveillance along
reasonable considering that Magallanes Street, Davao City.
unlike in the former, it was While they were within the
effected on the basis of a premises of the Rizal Memorial
probable cause. The probable Colleges they spotted petitioner
cause is that when the carrying a "buri" bag and they
petitioner acted suspiciously noticed him to be acting
and attempted to flee with the suspiciously. They approached the
buri bag there was a probable petitioner and identified
cause that he was concealing themselves as members of the INP.
something illegal in the bag Petitioner attempted to flee but his
and it was the right and duty of attempt to get away was thwarted
the police officers to inspect by the two notwithstanding his
the same.  resistance.They then checked the
"buri" bag of the petitioner where
they found one (1) caliber .38
Smith & Wesson revolver, two (2)
rounds of live ammunition for a .
38 caliber gun, a smoke (tear gas)
grenade, and two (2) live
ammunitions for a .22 caliber gun.
They brought the petitioner to the
police station for further
investigation and asked him to
show the necessary license or
authority to possess firearms and
ammunitions found in his
possession but he failed to do so. 
He was prosecuted for illegal
possession of firearms and
ammunitions in the RTC wherein
after a plea of not guilty.Petitioner
was found guilty of the offense
charged. 
PEOPLE v. Looking side to The Western Police District W/N Mengote can NO. There is no question that evidence obtained as a result of an
MENGOTE side and received a telephone call from an be held liable for illegal search or seizure is inadmissible in any proceeding for any
scratching informer that there were three illegal possession purpose. That is the absolute prohibition of Article III, Section 3(2),
abdomen suspicious looking persons at the of firearms. of the Constitution. This is the celebrated exclusionary rule based on
corner of Juan Luna and North Bay the justification given by Judge Learned Hand that "only in case the
Boulevard in Tondo, Manila. A prosecution, which itself controls the seizing officials, knows that it
surveillance team of cannot profit by their wrong will the wrong be repressed."
plainclothesmen was forthwith
dispatched to the place. The In cases falling under paragraphs (a) and (b) hereof, the person
patrolmen saw two men looking arrested without a warrant shall be forthwith delivered to the
from side to side, one of whom nearest police station or jail, and he shall be proceeded against in
holding his abdomen. They accordance with Rule 112, Section 7. We have carefully examined
approached the persons and the wording of this rule and cannot see how we can agree with the
identified themselves as prosecution. Par. (c) of Section 5 is obviously inapplicable as
policemen, whereupon the two Mengote was not an escapee from a penal institution when he was
tried to run but unable to escape arrested. We therefore confine ourselves to determining the
because the other lawmen lawfulness of his arrest under either Par. (a) or Par. (b) of this
surrounded them. The suspects section. Par. (a) requires that the person be arrested (1) after he has
were then searched. One of them committed or while he is actually committing or is at least
the accused-appellant was found attempting to commit an offense, (2) in the presence of the
with a .38 caliber with live arresting officer.
ammunitions in it, while his
companion had a fan knife. The
These requirements have not been established in the case at bar. At
weapons were taken from them
the time of the arrest in question, the accused- appellant was
and they were turned over to the
merely "looking from side to side" and "holding his abdomen,"
police headquarters for
according to the arresting officers themselves. There was apparently
investigation. An information was
no offense that had just been committed or was being actually
filed before the RTC convicting the
accused of illegal possession of
firearm arm. A witness testified committed or at least being attempted by Mengote in their
that the weapon was among the presence.
articles stolen at his shop, which
he reported to the police including
The case before us is different because there was nothing to
the revolver. For his part, Mengote
support the arresting officers' suspicion other than Mengote's
made no effort to prove that he
darting eyes and his hand on his abdomen. By no stretch of the
owned the fire arm or that he was
imagination could it have been inferred from these acts that an
licensed to possess it but instead,
offense had just been committed, or was actually being committed,
he claimed that the weapon was
or was at least being attempted in their presence.
planted on him at the time of his
arrest. He was convicted for
violation of P.D.1866 and was In short, there was no probable cause that, as the prosecution
sentenced to reclusion perpetua. incorrectly suggested, dispensed with the constitutional
In his appeal he pleads that the requirement of a warrant. Par. (b) is no less applicable because its
weapon was not admissible as no less stringent requirements have also not been satisfied. The
evidence against him because it prosecution has not shown that at the time of Mengote's arrest an
had been illegally seized and offense had in fact just been committed and that the arresting
therefore the fruit of a poisonous officers had personal knowledge of facts indicating that Mengote
tree. had committed it. All they had was hearsay information from the
telephone caller, and about a crime that had yet to be committed.
The truth is that they did not know then what offense, if at all, had
been committed and neither were they aware of the participation
therein of the accused- appellant. It was only later, after Danganan
had appeared at the police headquarters, that they learned of the
robbery in his house and of Mengote's supposed involvement
therein. 8 As for the illegal possession or the firearm found on
Mengote's person, the policemen discovered this only after he had
been searched and the investigation conducted later revealed that
he was not its owners nor was he licensed to possess it.
PEOPLE v. Racism It was held that vessels and Idel Aminnudin, accused-appellant W/N the seized NO. The present case presented no such urgency. From the
AMINNUDIN aircraft are subject to was arrested on June 25, 1984, effects may be conflicting declarations of the PC witnesses, it is clear that they had
warrantless searches and shortly after disembarking from used as evidence at least two days within which they could have obtained a warrant
seizures for violation of the the M/V Wilcon 9 at about 8:30 in as the search was to arrest and search Aminnudin. His name was known. The vehicle
customs law because these the evening, in Iloilo City. The PC allegedly an was identified. The date of its arrival was certain. And from the
vehicles may be quickly moved officers who were in fact waiting incident to a lawful information they had received, they could have persuaded a judge
out of the locality or for him because of a tip from one arrest that there was probable cause, indeed, to justify the issuance of a
jurisdiction before the warrant their informers simply accosted warrant. Yet they did nothing to comply Moreover, the accused
can be secured. him, inspected his bag and finding W/N the arrest appellant was not, at the moment of his arrest, committing a crime
what looked liked marijuana was legal nor was it shown that he was about to do so or that he had just
leaves took him to their done so.
headquarters for investigation.
The two bundles of suspect articles The accused- appellant was not, at the moment of his arrest,
were confiscated from him and committing a crime nor was it shown that he was about to do so or
later taken to the NBI laboratory that he had just done so. What he was doing was descending the
for examination. It was found to gangplank of the M/V Wilcon 9 and there was no outward
contain three kilos of what were indication that called for his arrest. To all appearances, he was like
later analyzed as marijuana leaves
by an NBI forensic examiner. An any of the other passengers innocently disembarking from the
information for violation of the vessel. It was only when the informer pointed to him as the carrier
Dangerous Drugs Act was filed of the marijuana that the suddenly became suspect and so subject
against him. Later, the information to apprehension. It was the furtive finger that triggered his arrest.
was amended to include Farida Ali The identification by the informer was the probable cause as
y Hassen, who had also been determined by the officers (and not a judge) that authorized them
arrested with him that same to pounce upon Aminnudin and immediately arrest him.
evening and likewise investigated.
Both were arraigned and pleaded
The search was not an incident of a lawful arrest because there was
not guilty. Subsequently, the fiscal
no warrant of arrest and the warrantless arrest did not come under
filed a motion to dismiss the
the exceptions allowed by the Rules of Court. Hence, the
charge against Ali on the basis of a
warrantless search was also illegal and the evidence obtained
sworn statement of the arresting
thereby was inadmissible.
officers absolving her after a
'thorough investigation." The
motion was granted, and trial
proceeded only against the
accused-appellant, who was
eventually convicted .

In his defense, Aminnudin


disclaimed the marijuana, averring
that all he had in his bag was his
clothing consisting of a jacket, two
shirts and two pairs of pants. He
alleged that he was arbitrarily
arrested and immediately
handcuffed. His bag was
confiscated without a search
warrant. At the PC headquarters,
he was manhandled to force him
to admit he was carrying the
marijuana, the investigator hitting
him with a piece of wood in the
chest and arms even as he parried
the blows while he was still
handcuffed. He insisted he did not
even know what marijuana looked
like and that his business was
selling watches and sometimes
cigarettes. However the RTC
rejected his allegations. Saying
that he only has two watches
during that time and that he did
not sufficiently proved the injuries
allegedly sustained.

You might also like